How to explain what's wrong with this application of the chain rule? The Next CEO of Stack OverflowEvaluating the reception of (epsilon, delta) definitionsHow does CalcChat work, and how can students who use it be encouraged to do so constructively?Should students be told they're wrongExactly what do I want these Calculus I students to learn?The purpose of mathematics in a liberal education when it is not a prerequisite to other subjectsHow can we neatly explain chain rule of differentiationTutoring a recalcitrant/awkward/exasperating student---special needs?Addressing fundamental math errorsHow to deal with “Why can't I just do …” in real analysis?Are questions on overlapping solids of revolutions without prior definitions and instructions fair given that there are divided interpretations?

How to pronounce fünf in 45

Cannot restore registry to default in Windows 10?

Is it possible to make a 9x9 table fit within the default margins?

Does Germany produce more waste than the US?

Salesforce opportunity stages

Why doesn't Shulchan Aruch include the laws of destroying fruit trees?

Is it a bad idea to plug the other end of ESD strap to wall ground?

Is it reasonable to ask other researchers to send me their previous grant applications?

How can a day be of 24 hours?

Traveling with my 5 year old daughter (as the father) without the mother from Germany to Mexico

How to implement Comparable so it is consistent with identity-equality

Prodigo = pro + ago?

How does a dynamic QR code work?

Free fall ellipse or parabola?

How exploitable/balanced is this homebrew spell: Spell Permanency?

Would a grinding machine be a simple and workable propulsion system for an interplanetary spacecraft?

How seriously should I take size and weight limits of hand luggage?

Do I need to write [sic] when including a quotation with a number less than 10 that isn't written out?

Raspberry pi 3 B with Ubuntu 18.04 server arm64: what pi version

What did the word "leisure" mean in late 18th Century usage?

Oldie but Goldie

Is a distribution that is normal, but highly skewed, considered Gaussian?

Is it correct to say moon starry nights?

Compensation for working overtime on Saturdays



How to explain what's wrong with this application of the chain rule?



The Next CEO of Stack OverflowEvaluating the reception of (epsilon, delta) definitionsHow does CalcChat work, and how can students who use it be encouraged to do so constructively?Should students be told they're wrongExactly what do I want these Calculus I students to learn?The purpose of mathematics in a liberal education when it is not a prerequisite to other subjectsHow can we neatly explain chain rule of differentiationTutoring a recalcitrant/awkward/exasperating student---special needs?Addressing fundamental math errorsHow to deal with “Why can't I just do …” in real analysis?Are questions on overlapping solids of revolutions without prior definitions and instructions fair given that there are divided interpretations?










11












$begingroup$


Yesterday a student in my calculus class attempted something like this:




Problem statement: Find the derivative of $3^(5x+1)$ with respect to $x$.




Proposed solution:



  • Let the inner function be given by $g(x)=3,$ and the outer by $f(z)=z^(5x+1)$, so that $$f(g(x))=3^(5x+1).$$


  • $f'(z)=(5x+1)cdot z^(5x)$ and $g'(x)=0$, so by the chain rule, $$fracdleft(3^(5x+1)right)dx = f'(g(x))g'(x)=0.$$

I had difficulties explaining what's wrong with this, and basically just told the student "the" right way to do it. Although I now have a rough idea of what's wrong, I'd like to hear from others:



  1. Have you seen similar attempts?

  2. How would you explain to a beginning calculus student what's wrong with this specific solution?









share|improve this question











$endgroup$







  • 6




    $begingroup$
    Nitpick: You mean differentiate, not derive (derive means deduce, or to to reach a conclusion using logical reasoning).
    $endgroup$
    – Andreas Rejbrand
    Mar 21 at 22:24






  • 4




    $begingroup$
    @MichaelBächtold While this would be true to the etymology of the words, I would caution than (at least) in British mathematical English derive from $f(x)$ wrt $x$ would be ungrammatical and not generally even understood. I would always write differentiate or find the derivative of.
    $endgroup$
    – dbmag9
    Mar 22 at 9:33







  • 1




    $begingroup$
    @dbmag9 I was sort of kidding. I'm not a native english speaker, but I'm aware that no mathematician would say "derive from". On the other hand: mathematicians seem to forget the etymology of words and write a lot of stuff which is ungrammatical in every language. Like calling $f$ a function of $x$ (when it's not) or saying stuff like "Consider the function $f(x)=x^2$" when they're actually considering an equation.
    $endgroup$
    – Michael Bächtold
    Mar 22 at 12:30







  • 3




    $begingroup$
    @MichaelBächtold: Indeed, I agree about that. I am known to be a bit pedantic, but I actually always write "consider the function $f$ defined by $f(x) = x^2$". Sometimes I even write "consider the function $f: mathbbR to mathbbR$ defined by $f(x) = x^2$ for every $x in mathbbR$". Since my main area of interest in mathematics is university-level mathematical education, I care a lot about language.
    $endgroup$
    – Andreas Rejbrand
    Mar 22 at 12:42







  • 2




    $begingroup$
    see math.stackexchange.com/questions/401122/… and the answers there for a similar issue
    $endgroup$
    – Matthew Towers
    Mar 22 at 12:48
















11












$begingroup$


Yesterday a student in my calculus class attempted something like this:




Problem statement: Find the derivative of $3^(5x+1)$ with respect to $x$.




Proposed solution:



  • Let the inner function be given by $g(x)=3,$ and the outer by $f(z)=z^(5x+1)$, so that $$f(g(x))=3^(5x+1).$$


  • $f'(z)=(5x+1)cdot z^(5x)$ and $g'(x)=0$, so by the chain rule, $$fracdleft(3^(5x+1)right)dx = f'(g(x))g'(x)=0.$$

I had difficulties explaining what's wrong with this, and basically just told the student "the" right way to do it. Although I now have a rough idea of what's wrong, I'd like to hear from others:



  1. Have you seen similar attempts?

  2. How would you explain to a beginning calculus student what's wrong with this specific solution?









share|improve this question











$endgroup$







  • 6




    $begingroup$
    Nitpick: You mean differentiate, not derive (derive means deduce, or to to reach a conclusion using logical reasoning).
    $endgroup$
    – Andreas Rejbrand
    Mar 21 at 22:24






  • 4




    $begingroup$
    @MichaelBächtold While this would be true to the etymology of the words, I would caution than (at least) in British mathematical English derive from $f(x)$ wrt $x$ would be ungrammatical and not generally even understood. I would always write differentiate or find the derivative of.
    $endgroup$
    – dbmag9
    Mar 22 at 9:33







  • 1




    $begingroup$
    @dbmag9 I was sort of kidding. I'm not a native english speaker, but I'm aware that no mathematician would say "derive from". On the other hand: mathematicians seem to forget the etymology of words and write a lot of stuff which is ungrammatical in every language. Like calling $f$ a function of $x$ (when it's not) or saying stuff like "Consider the function $f(x)=x^2$" when they're actually considering an equation.
    $endgroup$
    – Michael Bächtold
    Mar 22 at 12:30







  • 3




    $begingroup$
    @MichaelBächtold: Indeed, I agree about that. I am known to be a bit pedantic, but I actually always write "consider the function $f$ defined by $f(x) = x^2$". Sometimes I even write "consider the function $f: mathbbR to mathbbR$ defined by $f(x) = x^2$ for every $x in mathbbR$". Since my main area of interest in mathematics is university-level mathematical education, I care a lot about language.
    $endgroup$
    – Andreas Rejbrand
    Mar 22 at 12:42







  • 2




    $begingroup$
    see math.stackexchange.com/questions/401122/… and the answers there for a similar issue
    $endgroup$
    – Matthew Towers
    Mar 22 at 12:48














11












11








11


2



$begingroup$


Yesterday a student in my calculus class attempted something like this:




Problem statement: Find the derivative of $3^(5x+1)$ with respect to $x$.




Proposed solution:



  • Let the inner function be given by $g(x)=3,$ and the outer by $f(z)=z^(5x+1)$, so that $$f(g(x))=3^(5x+1).$$


  • $f'(z)=(5x+1)cdot z^(5x)$ and $g'(x)=0$, so by the chain rule, $$fracdleft(3^(5x+1)right)dx = f'(g(x))g'(x)=0.$$

I had difficulties explaining what's wrong with this, and basically just told the student "the" right way to do it. Although I now have a rough idea of what's wrong, I'd like to hear from others:



  1. Have you seen similar attempts?

  2. How would you explain to a beginning calculus student what's wrong with this specific solution?









share|improve this question











$endgroup$




Yesterday a student in my calculus class attempted something like this:




Problem statement: Find the derivative of $3^(5x+1)$ with respect to $x$.




Proposed solution:



  • Let the inner function be given by $g(x)=3,$ and the outer by $f(z)=z^(5x+1)$, so that $$f(g(x))=3^(5x+1).$$


  • $f'(z)=(5x+1)cdot z^(5x)$ and $g'(x)=0$, so by the chain rule, $$fracdleft(3^(5x+1)right)dx = f'(g(x))g'(x)=0.$$

I had difficulties explaining what's wrong with this, and basically just told the student "the" right way to do it. Although I now have a rough idea of what's wrong, I'd like to hear from others:



  1. Have you seen similar attempts?

  2. How would you explain to a beginning calculus student what's wrong with this specific solution?






calculus students-mistakes derivative






share|improve this question















share|improve this question













share|improve this question




share|improve this question








edited Mar 23 at 23:02









Namaste

16110




16110










asked Mar 21 at 16:25









Michael BächtoldMichael Bächtold

646515




646515







  • 6




    $begingroup$
    Nitpick: You mean differentiate, not derive (derive means deduce, or to to reach a conclusion using logical reasoning).
    $endgroup$
    – Andreas Rejbrand
    Mar 21 at 22:24






  • 4




    $begingroup$
    @MichaelBächtold While this would be true to the etymology of the words, I would caution than (at least) in British mathematical English derive from $f(x)$ wrt $x$ would be ungrammatical and not generally even understood. I would always write differentiate or find the derivative of.
    $endgroup$
    – dbmag9
    Mar 22 at 9:33







  • 1




    $begingroup$
    @dbmag9 I was sort of kidding. I'm not a native english speaker, but I'm aware that no mathematician would say "derive from". On the other hand: mathematicians seem to forget the etymology of words and write a lot of stuff which is ungrammatical in every language. Like calling $f$ a function of $x$ (when it's not) or saying stuff like "Consider the function $f(x)=x^2$" when they're actually considering an equation.
    $endgroup$
    – Michael Bächtold
    Mar 22 at 12:30







  • 3




    $begingroup$
    @MichaelBächtold: Indeed, I agree about that. I am known to be a bit pedantic, but I actually always write "consider the function $f$ defined by $f(x) = x^2$". Sometimes I even write "consider the function $f: mathbbR to mathbbR$ defined by $f(x) = x^2$ for every $x in mathbbR$". Since my main area of interest in mathematics is university-level mathematical education, I care a lot about language.
    $endgroup$
    – Andreas Rejbrand
    Mar 22 at 12:42







  • 2




    $begingroup$
    see math.stackexchange.com/questions/401122/… and the answers there for a similar issue
    $endgroup$
    – Matthew Towers
    Mar 22 at 12:48













  • 6




    $begingroup$
    Nitpick: You mean differentiate, not derive (derive means deduce, or to to reach a conclusion using logical reasoning).
    $endgroup$
    – Andreas Rejbrand
    Mar 21 at 22:24






  • 4




    $begingroup$
    @MichaelBächtold While this would be true to the etymology of the words, I would caution than (at least) in British mathematical English derive from $f(x)$ wrt $x$ would be ungrammatical and not generally even understood. I would always write differentiate or find the derivative of.
    $endgroup$
    – dbmag9
    Mar 22 at 9:33







  • 1




    $begingroup$
    @dbmag9 I was sort of kidding. I'm not a native english speaker, but I'm aware that no mathematician would say "derive from". On the other hand: mathematicians seem to forget the etymology of words and write a lot of stuff which is ungrammatical in every language. Like calling $f$ a function of $x$ (when it's not) or saying stuff like "Consider the function $f(x)=x^2$" when they're actually considering an equation.
    $endgroup$
    – Michael Bächtold
    Mar 22 at 12:30







  • 3




    $begingroup$
    @MichaelBächtold: Indeed, I agree about that. I am known to be a bit pedantic, but I actually always write "consider the function $f$ defined by $f(x) = x^2$". Sometimes I even write "consider the function $f: mathbbR to mathbbR$ defined by $f(x) = x^2$ for every $x in mathbbR$". Since my main area of interest in mathematics is university-level mathematical education, I care a lot about language.
    $endgroup$
    – Andreas Rejbrand
    Mar 22 at 12:42







  • 2




    $begingroup$
    see math.stackexchange.com/questions/401122/… and the answers there for a similar issue
    $endgroup$
    – Matthew Towers
    Mar 22 at 12:48








6




6




$begingroup$
Nitpick: You mean differentiate, not derive (derive means deduce, or to to reach a conclusion using logical reasoning).
$endgroup$
– Andreas Rejbrand
Mar 21 at 22:24




$begingroup$
Nitpick: You mean differentiate, not derive (derive means deduce, or to to reach a conclusion using logical reasoning).
$endgroup$
– Andreas Rejbrand
Mar 21 at 22:24




4




4




$begingroup$
@MichaelBächtold While this would be true to the etymology of the words, I would caution than (at least) in British mathematical English derive from $f(x)$ wrt $x$ would be ungrammatical and not generally even understood. I would always write differentiate or find the derivative of.
$endgroup$
– dbmag9
Mar 22 at 9:33





$begingroup$
@MichaelBächtold While this would be true to the etymology of the words, I would caution than (at least) in British mathematical English derive from $f(x)$ wrt $x$ would be ungrammatical and not generally even understood. I would always write differentiate or find the derivative of.
$endgroup$
– dbmag9
Mar 22 at 9:33





1




1




$begingroup$
@dbmag9 I was sort of kidding. I'm not a native english speaker, but I'm aware that no mathematician would say "derive from". On the other hand: mathematicians seem to forget the etymology of words and write a lot of stuff which is ungrammatical in every language. Like calling $f$ a function of $x$ (when it's not) or saying stuff like "Consider the function $f(x)=x^2$" when they're actually considering an equation.
$endgroup$
– Michael Bächtold
Mar 22 at 12:30





$begingroup$
@dbmag9 I was sort of kidding. I'm not a native english speaker, but I'm aware that no mathematician would say "derive from". On the other hand: mathematicians seem to forget the etymology of words and write a lot of stuff which is ungrammatical in every language. Like calling $f$ a function of $x$ (when it's not) or saying stuff like "Consider the function $f(x)=x^2$" when they're actually considering an equation.
$endgroup$
– Michael Bächtold
Mar 22 at 12:30





3




3




$begingroup$
@MichaelBächtold: Indeed, I agree about that. I am known to be a bit pedantic, but I actually always write "consider the function $f$ defined by $f(x) = x^2$". Sometimes I even write "consider the function $f: mathbbR to mathbbR$ defined by $f(x) = x^2$ for every $x in mathbbR$". Since my main area of interest in mathematics is university-level mathematical education, I care a lot about language.
$endgroup$
– Andreas Rejbrand
Mar 22 at 12:42





$begingroup$
@MichaelBächtold: Indeed, I agree about that. I am known to be a bit pedantic, but I actually always write "consider the function $f$ defined by $f(x) = x^2$". Sometimes I even write "consider the function $f: mathbbR to mathbbR$ defined by $f(x) = x^2$ for every $x in mathbbR$". Since my main area of interest in mathematics is university-level mathematical education, I care a lot about language.
$endgroup$
– Andreas Rejbrand
Mar 22 at 12:42





2




2




$begingroup$
see math.stackexchange.com/questions/401122/… and the answers there for a similar issue
$endgroup$
– Matthew Towers
Mar 22 at 12:48





$begingroup$
see math.stackexchange.com/questions/401122/… and the answers there for a similar issue
$endgroup$
– Matthew Towers
Mar 22 at 12:48











9 Answers
9






active

oldest

votes


















7












$begingroup$

$$
fracd (3^5x+1)dx = f'(g(x))g'(x)= fracd left(3^5x+1right)d(3) times fracd (3)dx.
$$



However $dfracd (3^5x+1)d(3)$ is undefined.






share|improve this answer











$endgroup$








  • 2




    $begingroup$
    Hence by definition, any substitution 𝑔(𝑥)=constant will blow up and fail for the same reason. (and that's before we get to the other issue with the substitution which the other answers point out: namely that we didn't really do a substitution, because there are still dangling references to x, as well as z)
    $endgroup$
    – smci
    Mar 23 at 15:05






  • 3




    $begingroup$
    This is not a valid explanation. Surely, the the chain rule notation works when $u=g(x)$ is a constant!
    $endgroup$
    – user52817
    Mar 23 at 16:01






  • 1




    $begingroup$
    @MichaelBächtold: It's ironic that you picked the totally wrong explanation on Math Educators SE. If you read a proper rigorous statement of the chain rule, it can be applied only under certain conditions, one of which is that the 'component' derivatives exist...
    $endgroup$
    – user21820
    Mar 24 at 7:50






  • 1




    $begingroup$
    @user21820 You just wrote 1/0 in a mathematical argument and the world didn't implode. But more seriously: Taemyr wrote $d(3^5x+1)/d3$ in order to point out to the student that it was undefined. I don't understand all the fuzz you are trying to make.
    $endgroup$
    – Michael Bächtold
    Mar 26 at 8:29






  • 1




    $begingroup$
    @Michael. User21820 is completely correct. This answer is wrong. I'm sorry to inform you that I'm worried that you are teaching students maths.
    $endgroup$
    – user7171
    yesterday


















21












$begingroup$

The root of the difficulty is that $x$ appears free in $f(z)$, but we are trying to "capture" it with $g(x)$, which is illegal. When we substitute $g(x)$ into $f(g(x))$, we have a variable clash:
$$
f(g(colorred x)) = 3^5colorbluex + 1
$$



The red (first) $x$ is a different variable from the blue (second) $x$. This is clearer if we rename the bound variable:
$$
f(g(colorred y)) = 3^5colorbluex + 1
$$



The original expression had $x$ bound to the $mathrm d x$, so by unbinding it, we have changed the meaning of the expression:
$$
fracmathrm dmathrm d colorblue x f(g(colorredy)) ne fracmathrm dmathrm d colorredy f(g(colorredy))
$$



(Incidentally, this is one reason I dislike the notation $f'(x)$, because it hides the variable of differentiation. Students must still be taught it, unfortunately, because Leibniz's notation is verbose in some contexts, but it should only be used as shorthand. Students should understand that it is a shorthand, and that there is still a variable of differentiation, even if it is not shown.)






share|improve this answer











$endgroup$








  • 1




    $begingroup$
    I like this answer, and agree with it in essence. But to make it precise one needs to be a bit more careful. For instance: the variabel $x$ is not really bound in the equation $h(x)=3^5x+1$. It would be bound if we wrote $forall xin mathbbRcolon h(x)=3^5x+1$, and then one might argue that variable capture happens in the composition. Alternatively we use mathematicians lambda calculus notation $h=(xmapsto 3^5x+1)$ and declare that $x$ is bound therein. Also: the matter of whether $dx$ binds $x$ is not so simple as you might think and (cont.)
    $endgroup$
    – Michael Bächtold
    Mar 22 at 8:36







  • 2




    $begingroup$
    (cont.) I don't quite understand what you don't like about $f'$, since in fact it needs to hide the name of the variable, since it is bound in $f$.
    $endgroup$
    – Michael Bächtold
    Mar 22 at 8:38






  • 1




    $begingroup$
    @Michael In common usage, $h(x) = 3^5x+1$ really does mean $h = (xmapsto 3^5x+1)$ most of the time, doesn’t it? It’s just the same widespread abuse of notation as writing “the function $h(x)$” when you really mean “the function $h$”, only milder. (Alternatively, you could say that it’s a different widespread abuse of notation, omitting universal quantification over apparently-unbound variables.)
    $endgroup$
    – Alex Shpilkin
    Mar 22 at 13:42











  • $begingroup$
    @AlexShpilkin I guess you are right. Still, I wouldn't explicitly encourage people to think that $exp(x)=e^x$ literally means the same as $exp=(xmapsto e^x)$, unless we also want them to conclude that $exp(0)=1$ means the same as $exp=(0mapsto 1)$.
    $endgroup$
    – Michael Bächtold
    Mar 22 at 16:34










  • $begingroup$
    @MichaelBächtold: I have no objection to $f'$, and in principle it ought to be OK to write $f'(x)$ if you can write $f'$. But $f'(x)$ is prone to the kind of error that your student made, unless you are also proposing to spend the extra class time to explain the difference between $f$ and $f(x)$ (because by default, your students will have no idea there is a difference). I would not recommend that at the introductory calculus level, however. It will likely shed more heat than light, and possibly leave your students more confused than they were to begin with.
    $endgroup$
    – Kevin
    Mar 22 at 16:43



















8












$begingroup$

f is not a function of (only) z - f here is a function of x as well as z. I think this explanation is intelligible to a calc 1 student, and gets at the heart of the matter.






share|improve this answer









$endgroup$








  • 2




    $begingroup$
    Hmm, so the student should reply: as soon as $f(x)$ contains parameters other than $x$ I am not allowed to apply the chain rule?
    $endgroup$
    – Michael Bächtold
    Mar 21 at 16:37











  • $begingroup$
    By the way: to my mind $f$ ist not a function of $z$ at all. Maybe you meant $f(z)$?
    $endgroup$
    – Michael Bächtold
    Mar 21 at 16:43






  • 1




    $begingroup$
    @MichaelBächtold: The student should know that if f(x) contains variables other than x then the chain rule doesn't apply. (This might be an opportunity to mention that there is a variant of the chain rule to be learned later for covering such situations.) I decline to get into a pedantic discussion of the distinction between f and f(z).
    $endgroup$
    – Henry Towsner
    Mar 21 at 18:14






  • 1




    $begingroup$
    I'm quite sure you use the chain rule to derive things that contain more than just $x$ in your calculus class, like $sqrtx^2+k$. You might say: that's ok if we treat $k$ as a constant and not as a variable. But then student might then ask: why am I not allowed to treat $x$ as a constant in the definition of $f$? (And a mathematician might add: what's the difference between a variable and a constant?). Apologies if my pedantry offends you.
    $endgroup$
    – Michael Bächtold
    Mar 21 at 18:49






  • 4




    $begingroup$
    @MichaelBächtold: In general, the difference between a variable and a constant is contextual and tricky to make precise, but for purposes of the chain rule in this case, x is a variable because we're taking the derivative with respect to it. I find that students don't usually have difficulty with this point (for instance, one could imagine students getting confused about the difference between the derivative of f(x)=c and f(x)=x, but that's not a particularly common issue), because it's a clean syntactic rule and it's backed up by the notion (x and z are conventionally variables, k isn't).
    $endgroup$
    – Henry Towsner
    Mar 21 at 19:28


















4












$begingroup$

This is a VERY VERY typical problem. In fact, it's a problem even for $fracddx3^x$, much less your example.



The way I try to deal with this is one of two ways.



  1. What has to happen first? To evaluate $3^5x+1$, you have to evaluate $5x+1$ first. So that is the inside function in the chain rule, just like in $sin(x^2)$ you have $x^2$ to evaluate first, so it is the inside function.


  2. You could rethink how we notate or talk about exponential functions. In particular, Excel has $e^x$ written as exp(x) (I think as an option). So one can ask what the "input" is here.


However, on the plus side the student does seem to have the chain rule down; it's just the exponential notation that is causing trouble. So there is definitely hope here. And again, you should not be surprised at encountering this, so it is worth your time to come up with several possible responses for it in the long run. Good luck!






share|improve this answer











$endgroup$




















    3












    $begingroup$

    This idea is fine, and you can use the multivariable chain rule to do it this way.



    Say we want to differentiate $h(x) = f(x)^g(x)$ with respect to $x$. Notice that we can write $h$ as the composite of $p: mathbbR to mathbbR^2$ defined by $p(t) = (f(t),g(t))$ with the function $E: mathbbR^2 to mathbbR$ defined by $E(u,v) = u^v$.



    By the multivariable chain rule,



    $$
    beginalign
    Dhbig|_x &= DEbig|_p(x) circ Dpbig|_x\
    &= left.beginbmatrix fracpartial Epartial u & fracpartial Epartial v endbmatrix right|_(u,v) = (f(x),g(x)) circ left.beginbmatrix fracpartial fpartial t \ fracpartial gpartial tendbmatrixright|_t = x\
    &= left.beginbmatrix vu^v-1 & ln(u) u^v endbmatrix right|_(u,v) = (f(x),g(x)) circ left.beginbmatrix f'(t) \ g'(t)endbmatrixright|_t = x\
    &= beginbmatrix g(x)(f(x))^g(x)-1 & ln(f(x)) (f(x))^g(x) endbmatrix circ beginbmatrix f'(x) \ g'(x)endbmatrix\
    &=f'(x)g(x)(f(x))^g(x)-1+g'(x)ln(x)f(x)^g(x)
    endalign
    $$



    Applying this to the problem in question, we see that $f'(x) =0$, so the first term disappears.



    So, in a sense, the student was trying to apply the multivariable chain rule (using the two variables $z$ and $x$), but didn't know how to do that yet. So you could tell them it is a good approach, but they will learn how to properly execute that approach in calc 3.






    share|improve this answer









    $endgroup$




















      1












      $begingroup$

      I had a similar problem with a student last week and could not succinctly explain why she could not select $e$ as the 'inner' function here: $$f(x) = e^8x +4$$



      The best explanation I have seen thus far, Paul's Notes, explains it in this way:




      Recall that the 'outside' function is the last operation that we would perform in an evaluation. In this case if we were to evaluate this function the last operation would be the exponential. Therefore, the outside function is the exponential function and the inside function is its exponent.







      share|improve this answer









      $endgroup$












      • $begingroup$
        Because 'e' is a constant? How would you explain it if instead of e, the base were '2'?
        $endgroup$
        – JoeTaxpayer
        Mar 22 at 15:27










      • $begingroup$
        Yes this is because 'e' is a constant. Therefore, the explanation would also work when the base is '2'.
        $endgroup$
        – MT Kop
        Mar 25 at 12:09










      • $begingroup$
        Agreed, that was what I was trying to suggest. The f(x) you shared has no ‘inner function’ .
        $endgroup$
        – JoeTaxpayer
        Mar 25 at 12:21


















      1












      $begingroup$

      The other answers have completely missed the mistake.
      $
      defrrmathbbR
      $



      Your student's error has nothing to do with exponentiatiation. Consider the following based on exactly the same error:




      $colorredtextLet (???)$ $f(y) = x$ and $g(x) = 1$.



      Then $1 = fracdxdx = (f∘g)'(x) = f'(g(x))·g'(x) = f'(1)·0 = 0$.




      The error lies in the very first line! It is extremely obvious once you actually attempt to make it rigorous. Recall that to define a function you must provide a domain as well as a rule that specifies the output for each input in the domain. And of course the rule has to be meaningful in the context where you want to define the function. So see what you get:




      $colorredtextLet (???)$ $f : rr→rr$ such that $f(y) = x$ for each $y∈rr$.



      Let $g : rr→rr$ such that $g(x) = 1$ for every $x∈rr$.




      The definition of $g$ is fine. The definition of $f$ is not fine! What on earth is $x$? The rule has to specify the output for each input $y∈rr$, so where did $x$ pop up from?



      As explained above, the error has nothing to do with differentiation. Rather, it is in the illegal definition of the function!






      share|improve this answer









      $endgroup$












      • $begingroup$
        Furthermore, it is misleading to bring in Leibniz notation when the question is about ordinary functions. Even if we do, the answer given by Taemyr is simply wrong. Given any real/complex variables $x,y,z$, the proper chain rule asserts $fracdzdx = fracdzdy ·fracdydx$ if $fracdzdy$ and $fracdydx$ are both defined. If you cannot prove that the two derivative expressions are defined, then you're not allowed to even write down the so-called chain rule, because it doesn't apply!
        $endgroup$
        – user21820
        Mar 24 at 8:17






      • 1




        $begingroup$
        I upvoted your answer, because it contributes something mathematically to the discussion, even if it completely lacks any tact. I would be curious to know in what sense you think my answer "completely misses the mark": it points out the same error as yours while being slightly more generous to the student (assuming that they were actually defining a function of two variables, rather than just writing something completely meaningless as you suppose). It then shows how the mistake (under this assumption) can be corrected through correct use of the multivariable chain rule.
        $endgroup$
        – Steven Gubkin
        Mar 24 at 14:24







      • 1




        $begingroup$
        (cont.) without going into free/bound variables, $alpha$-conversion and variable capture. That's what Kevin did. Not that I find it wrong to go into this stuff, but I don't think it's so useful for calculus students coming from engineering. Instead, telling them that $3^5x+1$ is not a function of $3$ and hence I cannot write $d3^5x+1/d3$ seems closer to what they'll need. It's a pity that a hundred years since Frege, logicians have not been able to properly formalise the surrounding notions of variables, constants, and functions of things.
        $endgroup$
        – Michael Bächtold
        Mar 24 at 20:45







      • 2




        $begingroup$
        @MichaelBächtold: I'll respond in chat. This is getting way too long for a comment thread.
        $endgroup$
        – user21820
        Mar 25 at 8:13






      • 1




        $begingroup$
        For future readers, the discussion is continued in depth starting from here.
        $endgroup$
        – user21820
        Mar 25 at 9:54


















      1












      $begingroup$

      Some users have expressed doubt at the validity of the accepted answer, so let me make it rigorous. To do that, we first need to make sense of the notation $fracdydx$. (Which is not a trivial task.)



      We start by changing perspective: instead of thinking of variables $x$ and $y$ as numbers, we think of them as smooth real valued functions on some manifold $M$. So $xcolon Mto mathbbR$ and $ycolon Mto mathbbR$. You might rightly ask: why would we do that and which manifold $M$ are you talking about? The answer to the first question is: because I'd like to talk about the differentials $dy$, $dx$ and say everyday stuff like "$y$ is a constant" or "$y$ is a function of $x$". All of that is impossible inside first order logic + ZFC if we simply interpret $x$ and $y$ as elements of $mathbbR$. Concerning the second question: think of $M$ as the physical state space underlying the problem we are trying to model and $x$ and $y$ as observables. If that sounds too unfamiliar: it's similar to how people in probability theory assume an underlying space of outcomes $Omega$, in order to talk about random variables. (Which are the things we really care about and historically came before $Omega$, just like $dx,dy$ historically came before manifolds, but I'm drifting of.)



      So, having fixed the background manifold $M$, whenever you hear me say variable, what I mean is a thing of type $Mto mathbbR$.




      Definition. Given two variables $x$ and $y$, call $y$ a function of $x$ if $dx$ is not zero almost everywhere and there exists a variable $qcolon M to mathbbR$ such that
      $$
      dy = q cdot dx.
      $$




      Intuitively, the equation $
      dy = q cdot dx
      $
      says that the change of $y$ is determined by the change of $x$, i.e. that $y$ depends on $x$.



      It's not hard to show that $q$ is uniquely determined by $x$ and $y$, hence we decide to denote it with $fracdydx$ and call it the derivative of $y$ wrt. $x$. It was originally called the differential coefficient, cause that's what it is.



      According to this definition, $3^5x+1$ is a function of $x$ (assuming $x$ is a true variabel, i.e. $dxneq 0$), but it's also a function of $5x+1$. On the other hand, $3^5x+1$ is not a function of $3$ since $d3=0$ (what we call a constant). In particular $fracd3^5x+1d3$ is undefined, as user21820 has been pointing out emphatically.



      We can now state the chain rule in Leibniz form




      Theorem. If $z$ is a function of $y$ and $y$ is a function of $x$, then $z$ is also a function $x$ and their differential coefficients satisfy
      $$
      fracdzdx=fracdzdycdot fracdydx
      $$




      The proof of this is trivial.



      From this perspective, what the student in my question was trying to do, was to let $z=3^5x+1$ and $y=3$. But the theorem does not apply, since $3^5x+1$ is not a function $3$. The same point of view can be used in the example discussed here, where a student attempted to differentiate $x^x$ by taking $x$ as inner function. Although that's allowed it just leads to
      $$
      fracdx^xdx=fracdx^xdxcdot fracdxdx
      $$

      which is of not much use.



      This is not to say that I don't appreciate the other answers (also users 21820). Taemyr's is just one of the three perspectives that haven been proposed. It might seem like it needs a lot of background to make it rigorous. But consider that mathematicians understood this stuff for at least 200 years without requiring manifolds to formalize it. And consider that the other approaches also require quite some background to make them rigorous (like quantifiers, variable bindings, the idea of dummy/bound variables etc. or derivatives of functions of multiple variables). Each has it advantages and disadvantages and none seems more right than the others, methinks.






      share|improve this answer









      $endgroup$












      • $begingroup$
        It seems you are finally agreeing with me that the accepted answer is not completely correct, because it literally wrote an equation involving what your answer states to be ill-defined. That from the beginning was my objection to it (see my first comment). I did not say that anything else was amiss with that answer, except that it fails to pinpoint the original student's error (regarding functions, not differentials).
        $endgroup$
        – user21820
        Mar 25 at 18:34











      • $begingroup$
        @user21820 You have a strange sense of humor. But thanks for making me laugh. Unfortunately you also made me give up my last hope of leading an honest conversation with you.
        $endgroup$
        – Michael Bächtold
        Mar 26 at 8:33










      • $begingroup$
        It's your choice, but you've been repeatedly misinterpreting whatever I say, and then blame me for it instead of considering that perhaps you are the one who is wrong. Your last comment insinuates that I am dishonest. That's false.
        $endgroup$
        – user21820
        Mar 26 at 9:05


















      1












      $begingroup$

      Applying the naive approach of a non-mathematician, to me the expression $z^(5x+1)$ points to a bivariate function,



      $$f(z,x) = z^(5x+1)$$



      (because "I see two variables in here"), and with $g(x) = 3$ we have defined



      $$f(g(x), x) = 3^(5x+1)$$



      Then



      $$frac df(g(x),x)dx = frac partial f(g(x),x)partial g(x)cdot frac dg(x)dx + frac partial f(g(x),x)partial xcdot frac dxdx$$



      $$=frac partial f(g(x),x)partial g(x)cdot 0 + frac partial f(g(x),x)partial xcdot 1 = frac partial f(g(x),x)partial x $$



      $$=frac partialpartial x left(3^(5x+1)right) $$



      This appears to be correct, although not useful, since we ended up back in the beginning. Am I doing something wrong here?






      share|improve this answer









      $endgroup$












      • $begingroup$
        It's unclear whether you know exactly what you're doing. You can't just say "I see two variables in here"! Given any real $x$, the function $f:mathbbR→mathbbR$ defined via $f(y) = x·y$ for $y∈mathbbR$ is a one-input function, not a bivariate function. Yes, the expression "$z^5x+1$" has two variables, so it is meaningful only in a context where both $x,z$ are defined (e.g. $x,z∈mathbbR$ and $z>0$), but that is precisely the true error in the asker's question (see my answer); it did not define $x$, and once you define $x$ you can't reuse it in defining $g$.
        $endgroup$
        – user21820
        Mar 27 at 7:49










      • $begingroup$
        Also, it is actually incorrect to write "$frac∂f(g(x),x)∂g(x)$". What you want is $left. frac∂f(t,x)∂t right|_t:=g(x)$. To prove that it is incorrect, consider that if you had wanted $fracd(f(g(x),g(x)))dx$ instead your 'proof' would have included the term "$frac∂f(g(x),g(x))∂g(x)$", which makes the mistake obvious.
        $endgroup$
        – user21820
        Mar 27 at 7:53











      • $begingroup$
        @user21820 On your first comment, let $x in mathbbR,; z>0$ and define the bivariate function $f(z,x) = z^(5x+1)$. Further, define $zequiv g(x)$ and also define $g(x) =3$ with $x$ defined as previously. Is there any problem with these definitions? This is not what the OP's student did of course, my answer was a reflection on why $f(z) = z^5x+1$ is not a correct expression, and that the moment you write $z^5x+1$ treating $z$ as a variable, what you can have is a bivariate function since $x$ is already defined (even if implicitly) as a variable.
        $endgroup$
        – Alecos Papadopoulos
        Mar 27 at 9:34










      • $begingroup$
        @user21820 Regarding your second comment, I was under the impression that in the left-hand side expression we include a variable of a function only once inside the parenthesis, since the $()$ in $f()$ just lists the variables of the function. When do we want or need to write something like $f(y,y)$, what purpose does it serve?
        $endgroup$
        – Alecos Papadopoulos
        Mar 27 at 9:49










      • $begingroup$
        You didn't understand my first comment; please read the second sentence again, which is a counter-example to your "I see two variables" thinking. You also don't seem to understand rigorous notation in your last comment. You say "the () in f() just lists the variables of the function", but you didn't even do that; you wrote "$f(g(x),x)$"! Furthermore, you suggest (falsely) that there is no purpose in writing something like "$f(y,y)$". No, it is not only legitimate but also very useful to be able to evaluate a two-input function along its 'diagonal'.
        $endgroup$
        – user21820
        Mar 27 at 11:25











      Your Answer





      StackExchange.ifUsing("editor", function ()
      return StackExchange.using("mathjaxEditing", function ()
      StackExchange.MarkdownEditor.creationCallbacks.add(function (editor, postfix)
      StackExchange.mathjaxEditing.prepareWmdForMathJax(editor, postfix, [["$", "$"], ["\\(","\\)"]]);
      );
      );
      , "mathjax-editing");

      StackExchange.ready(function()
      var channelOptions =
      tags: "".split(" "),
      id: "548"
      ;
      initTagRenderer("".split(" "), "".split(" "), channelOptions);

      StackExchange.using("externalEditor", function()
      // Have to fire editor after snippets, if snippets enabled
      if (StackExchange.settings.snippets.snippetsEnabled)
      StackExchange.using("snippets", function()
      createEditor();
      );

      else
      createEditor();

      );

      function createEditor()
      StackExchange.prepareEditor(
      heartbeatType: 'answer',
      autoActivateHeartbeat: false,
      convertImagesToLinks: false,
      noModals: true,
      showLowRepImageUploadWarning: true,
      reputationToPostImages: null,
      bindNavPrevention: true,
      postfix: "",
      imageUploader:
      brandingHtml: "Powered by u003ca class="icon-imgur-white" href="https://imgur.com/"u003eu003c/au003e",
      contentPolicyHtml: "User contributions licensed under u003ca href="https://creativecommons.org/licenses/by-sa/3.0/"u003ecc by-sa 3.0 with attribution requiredu003c/au003e u003ca href="https://stackoverflow.com/legal/content-policy"u003e(content policy)u003c/au003e",
      allowUrls: true
      ,
      noCode: true, onDemand: true,
      discardSelector: ".discard-answer"
      ,immediatelyShowMarkdownHelp:true
      );



      );













      draft saved

      draft discarded


















      StackExchange.ready(
      function ()
      StackExchange.openid.initPostLogin('.new-post-login', 'https%3a%2f%2fmatheducators.stackexchange.com%2fquestions%2f15366%2fhow-to-explain-whats-wrong-with-this-application-of-the-chain-rule%23new-answer', 'question_page');

      );

      Post as a guest















      Required, but never shown

























      9 Answers
      9






      active

      oldest

      votes








      9 Answers
      9






      active

      oldest

      votes









      active

      oldest

      votes






      active

      oldest

      votes









      7












      $begingroup$

      $$
      fracd (3^5x+1)dx = f'(g(x))g'(x)= fracd left(3^5x+1right)d(3) times fracd (3)dx.
      $$



      However $dfracd (3^5x+1)d(3)$ is undefined.






      share|improve this answer











      $endgroup$








      • 2




        $begingroup$
        Hence by definition, any substitution 𝑔(𝑥)=constant will blow up and fail for the same reason. (and that's before we get to the other issue with the substitution which the other answers point out: namely that we didn't really do a substitution, because there are still dangling references to x, as well as z)
        $endgroup$
        – smci
        Mar 23 at 15:05






      • 3




        $begingroup$
        This is not a valid explanation. Surely, the the chain rule notation works when $u=g(x)$ is a constant!
        $endgroup$
        – user52817
        Mar 23 at 16:01






      • 1




        $begingroup$
        @MichaelBächtold: It's ironic that you picked the totally wrong explanation on Math Educators SE. If you read a proper rigorous statement of the chain rule, it can be applied only under certain conditions, one of which is that the 'component' derivatives exist...
        $endgroup$
        – user21820
        Mar 24 at 7:50






      • 1




        $begingroup$
        @user21820 You just wrote 1/0 in a mathematical argument and the world didn't implode. But more seriously: Taemyr wrote $d(3^5x+1)/d3$ in order to point out to the student that it was undefined. I don't understand all the fuzz you are trying to make.
        $endgroup$
        – Michael Bächtold
        Mar 26 at 8:29






      • 1




        $begingroup$
        @Michael. User21820 is completely correct. This answer is wrong. I'm sorry to inform you that I'm worried that you are teaching students maths.
        $endgroup$
        – user7171
        yesterday















      7












      $begingroup$

      $$
      fracd (3^5x+1)dx = f'(g(x))g'(x)= fracd left(3^5x+1right)d(3) times fracd (3)dx.
      $$



      However $dfracd (3^5x+1)d(3)$ is undefined.






      share|improve this answer











      $endgroup$








      • 2




        $begingroup$
        Hence by definition, any substitution 𝑔(𝑥)=constant will blow up and fail for the same reason. (and that's before we get to the other issue with the substitution which the other answers point out: namely that we didn't really do a substitution, because there are still dangling references to x, as well as z)
        $endgroup$
        – smci
        Mar 23 at 15:05






      • 3




        $begingroup$
        This is not a valid explanation. Surely, the the chain rule notation works when $u=g(x)$ is a constant!
        $endgroup$
        – user52817
        Mar 23 at 16:01






      • 1




        $begingroup$
        @MichaelBächtold: It's ironic that you picked the totally wrong explanation on Math Educators SE. If you read a proper rigorous statement of the chain rule, it can be applied only under certain conditions, one of which is that the 'component' derivatives exist...
        $endgroup$
        – user21820
        Mar 24 at 7:50






      • 1




        $begingroup$
        @user21820 You just wrote 1/0 in a mathematical argument and the world didn't implode. But more seriously: Taemyr wrote $d(3^5x+1)/d3$ in order to point out to the student that it was undefined. I don't understand all the fuzz you are trying to make.
        $endgroup$
        – Michael Bächtold
        Mar 26 at 8:29






      • 1




        $begingroup$
        @Michael. User21820 is completely correct. This answer is wrong. I'm sorry to inform you that I'm worried that you are teaching students maths.
        $endgroup$
        – user7171
        yesterday













      7












      7








      7





      $begingroup$

      $$
      fracd (3^5x+1)dx = f'(g(x))g'(x)= fracd left(3^5x+1right)d(3) times fracd (3)dx.
      $$



      However $dfracd (3^5x+1)d(3)$ is undefined.






      share|improve this answer











      $endgroup$



      $$
      fracd (3^5x+1)dx = f'(g(x))g'(x)= fracd left(3^5x+1right)d(3) times fracd (3)dx.
      $$



      However $dfracd (3^5x+1)d(3)$ is undefined.







      share|improve this answer














      share|improve this answer



      share|improve this answer








      edited Mar 23 at 0:34









      Namaste

      16110




      16110










      answered Mar 22 at 11:14









      TaemyrTaemyr

      21012




      21012







      • 2




        $begingroup$
        Hence by definition, any substitution 𝑔(𝑥)=constant will blow up and fail for the same reason. (and that's before we get to the other issue with the substitution which the other answers point out: namely that we didn't really do a substitution, because there are still dangling references to x, as well as z)
        $endgroup$
        – smci
        Mar 23 at 15:05






      • 3




        $begingroup$
        This is not a valid explanation. Surely, the the chain rule notation works when $u=g(x)$ is a constant!
        $endgroup$
        – user52817
        Mar 23 at 16:01






      • 1




        $begingroup$
        @MichaelBächtold: It's ironic that you picked the totally wrong explanation on Math Educators SE. If you read a proper rigorous statement of the chain rule, it can be applied only under certain conditions, one of which is that the 'component' derivatives exist...
        $endgroup$
        – user21820
        Mar 24 at 7:50






      • 1




        $begingroup$
        @user21820 You just wrote 1/0 in a mathematical argument and the world didn't implode. But more seriously: Taemyr wrote $d(3^5x+1)/d3$ in order to point out to the student that it was undefined. I don't understand all the fuzz you are trying to make.
        $endgroup$
        – Michael Bächtold
        Mar 26 at 8:29






      • 1




        $begingroup$
        @Michael. User21820 is completely correct. This answer is wrong. I'm sorry to inform you that I'm worried that you are teaching students maths.
        $endgroup$
        – user7171
        yesterday












      • 2




        $begingroup$
        Hence by definition, any substitution 𝑔(𝑥)=constant will blow up and fail for the same reason. (and that's before we get to the other issue with the substitution which the other answers point out: namely that we didn't really do a substitution, because there are still dangling references to x, as well as z)
        $endgroup$
        – smci
        Mar 23 at 15:05






      • 3




        $begingroup$
        This is not a valid explanation. Surely, the the chain rule notation works when $u=g(x)$ is a constant!
        $endgroup$
        – user52817
        Mar 23 at 16:01






      • 1




        $begingroup$
        @MichaelBächtold: It's ironic that you picked the totally wrong explanation on Math Educators SE. If you read a proper rigorous statement of the chain rule, it can be applied only under certain conditions, one of which is that the 'component' derivatives exist...
        $endgroup$
        – user21820
        Mar 24 at 7:50






      • 1




        $begingroup$
        @user21820 You just wrote 1/0 in a mathematical argument and the world didn't implode. But more seriously: Taemyr wrote $d(3^5x+1)/d3$ in order to point out to the student that it was undefined. I don't understand all the fuzz you are trying to make.
        $endgroup$
        – Michael Bächtold
        Mar 26 at 8:29






      • 1




        $begingroup$
        @Michael. User21820 is completely correct. This answer is wrong. I'm sorry to inform you that I'm worried that you are teaching students maths.
        $endgroup$
        – user7171
        yesterday







      2




      2




      $begingroup$
      Hence by definition, any substitution 𝑔(𝑥)=constant will blow up and fail for the same reason. (and that's before we get to the other issue with the substitution which the other answers point out: namely that we didn't really do a substitution, because there are still dangling references to x, as well as z)
      $endgroup$
      – smci
      Mar 23 at 15:05




      $begingroup$
      Hence by definition, any substitution 𝑔(𝑥)=constant will blow up and fail for the same reason. (and that's before we get to the other issue with the substitution which the other answers point out: namely that we didn't really do a substitution, because there are still dangling references to x, as well as z)
      $endgroup$
      – smci
      Mar 23 at 15:05




      3




      3




      $begingroup$
      This is not a valid explanation. Surely, the the chain rule notation works when $u=g(x)$ is a constant!
      $endgroup$
      – user52817
      Mar 23 at 16:01




      $begingroup$
      This is not a valid explanation. Surely, the the chain rule notation works when $u=g(x)$ is a constant!
      $endgroup$
      – user52817
      Mar 23 at 16:01




      1




      1




      $begingroup$
      @MichaelBächtold: It's ironic that you picked the totally wrong explanation on Math Educators SE. If you read a proper rigorous statement of the chain rule, it can be applied only under certain conditions, one of which is that the 'component' derivatives exist...
      $endgroup$
      – user21820
      Mar 24 at 7:50




      $begingroup$
      @MichaelBächtold: It's ironic that you picked the totally wrong explanation on Math Educators SE. If you read a proper rigorous statement of the chain rule, it can be applied only under certain conditions, one of which is that the 'component' derivatives exist...
      $endgroup$
      – user21820
      Mar 24 at 7:50




      1




      1




      $begingroup$
      @user21820 You just wrote 1/0 in a mathematical argument and the world didn't implode. But more seriously: Taemyr wrote $d(3^5x+1)/d3$ in order to point out to the student that it was undefined. I don't understand all the fuzz you are trying to make.
      $endgroup$
      – Michael Bächtold
      Mar 26 at 8:29




      $begingroup$
      @user21820 You just wrote 1/0 in a mathematical argument and the world didn't implode. But more seriously: Taemyr wrote $d(3^5x+1)/d3$ in order to point out to the student that it was undefined. I don't understand all the fuzz you are trying to make.
      $endgroup$
      – Michael Bächtold
      Mar 26 at 8:29




      1




      1




      $begingroup$
      @Michael. User21820 is completely correct. This answer is wrong. I'm sorry to inform you that I'm worried that you are teaching students maths.
      $endgroup$
      – user7171
      yesterday




      $begingroup$
      @Michael. User21820 is completely correct. This answer is wrong. I'm sorry to inform you that I'm worried that you are teaching students maths.
      $endgroup$
      – user7171
      yesterday











      21












      $begingroup$

      The root of the difficulty is that $x$ appears free in $f(z)$, but we are trying to "capture" it with $g(x)$, which is illegal. When we substitute $g(x)$ into $f(g(x))$, we have a variable clash:
      $$
      f(g(colorred x)) = 3^5colorbluex + 1
      $$



      The red (first) $x$ is a different variable from the blue (second) $x$. This is clearer if we rename the bound variable:
      $$
      f(g(colorred y)) = 3^5colorbluex + 1
      $$



      The original expression had $x$ bound to the $mathrm d x$, so by unbinding it, we have changed the meaning of the expression:
      $$
      fracmathrm dmathrm d colorblue x f(g(colorredy)) ne fracmathrm dmathrm d colorredy f(g(colorredy))
      $$



      (Incidentally, this is one reason I dislike the notation $f'(x)$, because it hides the variable of differentiation. Students must still be taught it, unfortunately, because Leibniz's notation is verbose in some contexts, but it should only be used as shorthand. Students should understand that it is a shorthand, and that there is still a variable of differentiation, even if it is not shown.)






      share|improve this answer











      $endgroup$








      • 1




        $begingroup$
        I like this answer, and agree with it in essence. But to make it precise one needs to be a bit more careful. For instance: the variabel $x$ is not really bound in the equation $h(x)=3^5x+1$. It would be bound if we wrote $forall xin mathbbRcolon h(x)=3^5x+1$, and then one might argue that variable capture happens in the composition. Alternatively we use mathematicians lambda calculus notation $h=(xmapsto 3^5x+1)$ and declare that $x$ is bound therein. Also: the matter of whether $dx$ binds $x$ is not so simple as you might think and (cont.)
        $endgroup$
        – Michael Bächtold
        Mar 22 at 8:36







      • 2




        $begingroup$
        (cont.) I don't quite understand what you don't like about $f'$, since in fact it needs to hide the name of the variable, since it is bound in $f$.
        $endgroup$
        – Michael Bächtold
        Mar 22 at 8:38






      • 1




        $begingroup$
        @Michael In common usage, $h(x) = 3^5x+1$ really does mean $h = (xmapsto 3^5x+1)$ most of the time, doesn’t it? It’s just the same widespread abuse of notation as writing “the function $h(x)$” when you really mean “the function $h$”, only milder. (Alternatively, you could say that it’s a different widespread abuse of notation, omitting universal quantification over apparently-unbound variables.)
        $endgroup$
        – Alex Shpilkin
        Mar 22 at 13:42











      • $begingroup$
        @AlexShpilkin I guess you are right. Still, I wouldn't explicitly encourage people to think that $exp(x)=e^x$ literally means the same as $exp=(xmapsto e^x)$, unless we also want them to conclude that $exp(0)=1$ means the same as $exp=(0mapsto 1)$.
        $endgroup$
        – Michael Bächtold
        Mar 22 at 16:34










      • $begingroup$
        @MichaelBächtold: I have no objection to $f'$, and in principle it ought to be OK to write $f'(x)$ if you can write $f'$. But $f'(x)$ is prone to the kind of error that your student made, unless you are also proposing to spend the extra class time to explain the difference between $f$ and $f(x)$ (because by default, your students will have no idea there is a difference). I would not recommend that at the introductory calculus level, however. It will likely shed more heat than light, and possibly leave your students more confused than they were to begin with.
        $endgroup$
        – Kevin
        Mar 22 at 16:43
















      21












      $begingroup$

      The root of the difficulty is that $x$ appears free in $f(z)$, but we are trying to "capture" it with $g(x)$, which is illegal. When we substitute $g(x)$ into $f(g(x))$, we have a variable clash:
      $$
      f(g(colorred x)) = 3^5colorbluex + 1
      $$



      The red (first) $x$ is a different variable from the blue (second) $x$. This is clearer if we rename the bound variable:
      $$
      f(g(colorred y)) = 3^5colorbluex + 1
      $$



      The original expression had $x$ bound to the $mathrm d x$, so by unbinding it, we have changed the meaning of the expression:
      $$
      fracmathrm dmathrm d colorblue x f(g(colorredy)) ne fracmathrm dmathrm d colorredy f(g(colorredy))
      $$



      (Incidentally, this is one reason I dislike the notation $f'(x)$, because it hides the variable of differentiation. Students must still be taught it, unfortunately, because Leibniz's notation is verbose in some contexts, but it should only be used as shorthand. Students should understand that it is a shorthand, and that there is still a variable of differentiation, even if it is not shown.)






      share|improve this answer











      $endgroup$








      • 1




        $begingroup$
        I like this answer, and agree with it in essence. But to make it precise one needs to be a bit more careful. For instance: the variabel $x$ is not really bound in the equation $h(x)=3^5x+1$. It would be bound if we wrote $forall xin mathbbRcolon h(x)=3^5x+1$, and then one might argue that variable capture happens in the composition. Alternatively we use mathematicians lambda calculus notation $h=(xmapsto 3^5x+1)$ and declare that $x$ is bound therein. Also: the matter of whether $dx$ binds $x$ is not so simple as you might think and (cont.)
        $endgroup$
        – Michael Bächtold
        Mar 22 at 8:36







      • 2




        $begingroup$
        (cont.) I don't quite understand what you don't like about $f'$, since in fact it needs to hide the name of the variable, since it is bound in $f$.
        $endgroup$
        – Michael Bächtold
        Mar 22 at 8:38






      • 1




        $begingroup$
        @Michael In common usage, $h(x) = 3^5x+1$ really does mean $h = (xmapsto 3^5x+1)$ most of the time, doesn’t it? It’s just the same widespread abuse of notation as writing “the function $h(x)$” when you really mean “the function $h$”, only milder. (Alternatively, you could say that it’s a different widespread abuse of notation, omitting universal quantification over apparently-unbound variables.)
        $endgroup$
        – Alex Shpilkin
        Mar 22 at 13:42











      • $begingroup$
        @AlexShpilkin I guess you are right. Still, I wouldn't explicitly encourage people to think that $exp(x)=e^x$ literally means the same as $exp=(xmapsto e^x)$, unless we also want them to conclude that $exp(0)=1$ means the same as $exp=(0mapsto 1)$.
        $endgroup$
        – Michael Bächtold
        Mar 22 at 16:34










      • $begingroup$
        @MichaelBächtold: I have no objection to $f'$, and in principle it ought to be OK to write $f'(x)$ if you can write $f'$. But $f'(x)$ is prone to the kind of error that your student made, unless you are also proposing to spend the extra class time to explain the difference between $f$ and $f(x)$ (because by default, your students will have no idea there is a difference). I would not recommend that at the introductory calculus level, however. It will likely shed more heat than light, and possibly leave your students more confused than they were to begin with.
        $endgroup$
        – Kevin
        Mar 22 at 16:43














      21












      21








      21





      $begingroup$

      The root of the difficulty is that $x$ appears free in $f(z)$, but we are trying to "capture" it with $g(x)$, which is illegal. When we substitute $g(x)$ into $f(g(x))$, we have a variable clash:
      $$
      f(g(colorred x)) = 3^5colorbluex + 1
      $$



      The red (first) $x$ is a different variable from the blue (second) $x$. This is clearer if we rename the bound variable:
      $$
      f(g(colorred y)) = 3^5colorbluex + 1
      $$



      The original expression had $x$ bound to the $mathrm d x$, so by unbinding it, we have changed the meaning of the expression:
      $$
      fracmathrm dmathrm d colorblue x f(g(colorredy)) ne fracmathrm dmathrm d colorredy f(g(colorredy))
      $$



      (Incidentally, this is one reason I dislike the notation $f'(x)$, because it hides the variable of differentiation. Students must still be taught it, unfortunately, because Leibniz's notation is verbose in some contexts, but it should only be used as shorthand. Students should understand that it is a shorthand, and that there is still a variable of differentiation, even if it is not shown.)






      share|improve this answer











      $endgroup$



      The root of the difficulty is that $x$ appears free in $f(z)$, but we are trying to "capture" it with $g(x)$, which is illegal. When we substitute $g(x)$ into $f(g(x))$, we have a variable clash:
      $$
      f(g(colorred x)) = 3^5colorbluex + 1
      $$



      The red (first) $x$ is a different variable from the blue (second) $x$. This is clearer if we rename the bound variable:
      $$
      f(g(colorred y)) = 3^5colorbluex + 1
      $$



      The original expression had $x$ bound to the $mathrm d x$, so by unbinding it, we have changed the meaning of the expression:
      $$
      fracmathrm dmathrm d colorblue x f(g(colorredy)) ne fracmathrm dmathrm d colorredy f(g(colorredy))
      $$



      (Incidentally, this is one reason I dislike the notation $f'(x)$, because it hides the variable of differentiation. Students must still be taught it, unfortunately, because Leibniz's notation is verbose in some contexts, but it should only be used as shorthand. Students should understand that it is a shorthand, and that there is still a variable of differentiation, even if it is not shown.)







      share|improve this answer














      share|improve this answer



      share|improve this answer








      edited Mar 21 at 21:22

























      answered Mar 21 at 21:17









      KevinKevin

      36616




      36616







      • 1




        $begingroup$
        I like this answer, and agree with it in essence. But to make it precise one needs to be a bit more careful. For instance: the variabel $x$ is not really bound in the equation $h(x)=3^5x+1$. It would be bound if we wrote $forall xin mathbbRcolon h(x)=3^5x+1$, and then one might argue that variable capture happens in the composition. Alternatively we use mathematicians lambda calculus notation $h=(xmapsto 3^5x+1)$ and declare that $x$ is bound therein. Also: the matter of whether $dx$ binds $x$ is not so simple as you might think and (cont.)
        $endgroup$
        – Michael Bächtold
        Mar 22 at 8:36







      • 2




        $begingroup$
        (cont.) I don't quite understand what you don't like about $f'$, since in fact it needs to hide the name of the variable, since it is bound in $f$.
        $endgroup$
        – Michael Bächtold
        Mar 22 at 8:38






      • 1




        $begingroup$
        @Michael In common usage, $h(x) = 3^5x+1$ really does mean $h = (xmapsto 3^5x+1)$ most of the time, doesn’t it? It’s just the same widespread abuse of notation as writing “the function $h(x)$” when you really mean “the function $h$”, only milder. (Alternatively, you could say that it’s a different widespread abuse of notation, omitting universal quantification over apparently-unbound variables.)
        $endgroup$
        – Alex Shpilkin
        Mar 22 at 13:42











      • $begingroup$
        @AlexShpilkin I guess you are right. Still, I wouldn't explicitly encourage people to think that $exp(x)=e^x$ literally means the same as $exp=(xmapsto e^x)$, unless we also want them to conclude that $exp(0)=1$ means the same as $exp=(0mapsto 1)$.
        $endgroup$
        – Michael Bächtold
        Mar 22 at 16:34










      • $begingroup$
        @MichaelBächtold: I have no objection to $f'$, and in principle it ought to be OK to write $f'(x)$ if you can write $f'$. But $f'(x)$ is prone to the kind of error that your student made, unless you are also proposing to spend the extra class time to explain the difference between $f$ and $f(x)$ (because by default, your students will have no idea there is a difference). I would not recommend that at the introductory calculus level, however. It will likely shed more heat than light, and possibly leave your students more confused than they were to begin with.
        $endgroup$
        – Kevin
        Mar 22 at 16:43













      • 1




        $begingroup$
        I like this answer, and agree with it in essence. But to make it precise one needs to be a bit more careful. For instance: the variabel $x$ is not really bound in the equation $h(x)=3^5x+1$. It would be bound if we wrote $forall xin mathbbRcolon h(x)=3^5x+1$, and then one might argue that variable capture happens in the composition. Alternatively we use mathematicians lambda calculus notation $h=(xmapsto 3^5x+1)$ and declare that $x$ is bound therein. Also: the matter of whether $dx$ binds $x$ is not so simple as you might think and (cont.)
        $endgroup$
        – Michael Bächtold
        Mar 22 at 8:36







      • 2




        $begingroup$
        (cont.) I don't quite understand what you don't like about $f'$, since in fact it needs to hide the name of the variable, since it is bound in $f$.
        $endgroup$
        – Michael Bächtold
        Mar 22 at 8:38






      • 1




        $begingroup$
        @Michael In common usage, $h(x) = 3^5x+1$ really does mean $h = (xmapsto 3^5x+1)$ most of the time, doesn’t it? It’s just the same widespread abuse of notation as writing “the function $h(x)$” when you really mean “the function $h$”, only milder. (Alternatively, you could say that it’s a different widespread abuse of notation, omitting universal quantification over apparently-unbound variables.)
        $endgroup$
        – Alex Shpilkin
        Mar 22 at 13:42











      • $begingroup$
        @AlexShpilkin I guess you are right. Still, I wouldn't explicitly encourage people to think that $exp(x)=e^x$ literally means the same as $exp=(xmapsto e^x)$, unless we also want them to conclude that $exp(0)=1$ means the same as $exp=(0mapsto 1)$.
        $endgroup$
        – Michael Bächtold
        Mar 22 at 16:34










      • $begingroup$
        @MichaelBächtold: I have no objection to $f'$, and in principle it ought to be OK to write $f'(x)$ if you can write $f'$. But $f'(x)$ is prone to the kind of error that your student made, unless you are also proposing to spend the extra class time to explain the difference between $f$ and $f(x)$ (because by default, your students will have no idea there is a difference). I would not recommend that at the introductory calculus level, however. It will likely shed more heat than light, and possibly leave your students more confused than they were to begin with.
        $endgroup$
        – Kevin
        Mar 22 at 16:43








      1




      1




      $begingroup$
      I like this answer, and agree with it in essence. But to make it precise one needs to be a bit more careful. For instance: the variabel $x$ is not really bound in the equation $h(x)=3^5x+1$. It would be bound if we wrote $forall xin mathbbRcolon h(x)=3^5x+1$, and then one might argue that variable capture happens in the composition. Alternatively we use mathematicians lambda calculus notation $h=(xmapsto 3^5x+1)$ and declare that $x$ is bound therein. Also: the matter of whether $dx$ binds $x$ is not so simple as you might think and (cont.)
      $endgroup$
      – Michael Bächtold
      Mar 22 at 8:36





      $begingroup$
      I like this answer, and agree with it in essence. But to make it precise one needs to be a bit more careful. For instance: the variabel $x$ is not really bound in the equation $h(x)=3^5x+1$. It would be bound if we wrote $forall xin mathbbRcolon h(x)=3^5x+1$, and then one might argue that variable capture happens in the composition. Alternatively we use mathematicians lambda calculus notation $h=(xmapsto 3^5x+1)$ and declare that $x$ is bound therein. Also: the matter of whether $dx$ binds $x$ is not so simple as you might think and (cont.)
      $endgroup$
      – Michael Bächtold
      Mar 22 at 8:36





      2




      2




      $begingroup$
      (cont.) I don't quite understand what you don't like about $f'$, since in fact it needs to hide the name of the variable, since it is bound in $f$.
      $endgroup$
      – Michael Bächtold
      Mar 22 at 8:38




      $begingroup$
      (cont.) I don't quite understand what you don't like about $f'$, since in fact it needs to hide the name of the variable, since it is bound in $f$.
      $endgroup$
      – Michael Bächtold
      Mar 22 at 8:38




      1




      1




      $begingroup$
      @Michael In common usage, $h(x) = 3^5x+1$ really does mean $h = (xmapsto 3^5x+1)$ most of the time, doesn’t it? It’s just the same widespread abuse of notation as writing “the function $h(x)$” when you really mean “the function $h$”, only milder. (Alternatively, you could say that it’s a different widespread abuse of notation, omitting universal quantification over apparently-unbound variables.)
      $endgroup$
      – Alex Shpilkin
      Mar 22 at 13:42





      $begingroup$
      @Michael In common usage, $h(x) = 3^5x+1$ really does mean $h = (xmapsto 3^5x+1)$ most of the time, doesn’t it? It’s just the same widespread abuse of notation as writing “the function $h(x)$” when you really mean “the function $h$”, only milder. (Alternatively, you could say that it’s a different widespread abuse of notation, omitting universal quantification over apparently-unbound variables.)
      $endgroup$
      – Alex Shpilkin
      Mar 22 at 13:42













      $begingroup$
      @AlexShpilkin I guess you are right. Still, I wouldn't explicitly encourage people to think that $exp(x)=e^x$ literally means the same as $exp=(xmapsto e^x)$, unless we also want them to conclude that $exp(0)=1$ means the same as $exp=(0mapsto 1)$.
      $endgroup$
      – Michael Bächtold
      Mar 22 at 16:34




      $begingroup$
      @AlexShpilkin I guess you are right. Still, I wouldn't explicitly encourage people to think that $exp(x)=e^x$ literally means the same as $exp=(xmapsto e^x)$, unless we also want them to conclude that $exp(0)=1$ means the same as $exp=(0mapsto 1)$.
      $endgroup$
      – Michael Bächtold
      Mar 22 at 16:34












      $begingroup$
      @MichaelBächtold: I have no objection to $f'$, and in principle it ought to be OK to write $f'(x)$ if you can write $f'$. But $f'(x)$ is prone to the kind of error that your student made, unless you are also proposing to spend the extra class time to explain the difference between $f$ and $f(x)$ (because by default, your students will have no idea there is a difference). I would not recommend that at the introductory calculus level, however. It will likely shed more heat than light, and possibly leave your students more confused than they were to begin with.
      $endgroup$
      – Kevin
      Mar 22 at 16:43





      $begingroup$
      @MichaelBächtold: I have no objection to $f'$, and in principle it ought to be OK to write $f'(x)$ if you can write $f'$. But $f'(x)$ is prone to the kind of error that your student made, unless you are also proposing to spend the extra class time to explain the difference between $f$ and $f(x)$ (because by default, your students will have no idea there is a difference). I would not recommend that at the introductory calculus level, however. It will likely shed more heat than light, and possibly leave your students more confused than they were to begin with.
      $endgroup$
      – Kevin
      Mar 22 at 16:43












      8












      $begingroup$

      f is not a function of (only) z - f here is a function of x as well as z. I think this explanation is intelligible to a calc 1 student, and gets at the heart of the matter.






      share|improve this answer









      $endgroup$








      • 2




        $begingroup$
        Hmm, so the student should reply: as soon as $f(x)$ contains parameters other than $x$ I am not allowed to apply the chain rule?
        $endgroup$
        – Michael Bächtold
        Mar 21 at 16:37











      • $begingroup$
        By the way: to my mind $f$ ist not a function of $z$ at all. Maybe you meant $f(z)$?
        $endgroup$
        – Michael Bächtold
        Mar 21 at 16:43






      • 1




        $begingroup$
        @MichaelBächtold: The student should know that if f(x) contains variables other than x then the chain rule doesn't apply. (This might be an opportunity to mention that there is a variant of the chain rule to be learned later for covering such situations.) I decline to get into a pedantic discussion of the distinction between f and f(z).
        $endgroup$
        – Henry Towsner
        Mar 21 at 18:14






      • 1




        $begingroup$
        I'm quite sure you use the chain rule to derive things that contain more than just $x$ in your calculus class, like $sqrtx^2+k$. You might say: that's ok if we treat $k$ as a constant and not as a variable. But then student might then ask: why am I not allowed to treat $x$ as a constant in the definition of $f$? (And a mathematician might add: what's the difference between a variable and a constant?). Apologies if my pedantry offends you.
        $endgroup$
        – Michael Bächtold
        Mar 21 at 18:49






      • 4




        $begingroup$
        @MichaelBächtold: In general, the difference between a variable and a constant is contextual and tricky to make precise, but for purposes of the chain rule in this case, x is a variable because we're taking the derivative with respect to it. I find that students don't usually have difficulty with this point (for instance, one could imagine students getting confused about the difference between the derivative of f(x)=c and f(x)=x, but that's not a particularly common issue), because it's a clean syntactic rule and it's backed up by the notion (x and z are conventionally variables, k isn't).
        $endgroup$
        – Henry Towsner
        Mar 21 at 19:28















      8












      $begingroup$

      f is not a function of (only) z - f here is a function of x as well as z. I think this explanation is intelligible to a calc 1 student, and gets at the heart of the matter.






      share|improve this answer









      $endgroup$








      • 2




        $begingroup$
        Hmm, so the student should reply: as soon as $f(x)$ contains parameters other than $x$ I am not allowed to apply the chain rule?
        $endgroup$
        – Michael Bächtold
        Mar 21 at 16:37











      • $begingroup$
        By the way: to my mind $f$ ist not a function of $z$ at all. Maybe you meant $f(z)$?
        $endgroup$
        – Michael Bächtold
        Mar 21 at 16:43






      • 1




        $begingroup$
        @MichaelBächtold: The student should know that if f(x) contains variables other than x then the chain rule doesn't apply. (This might be an opportunity to mention that there is a variant of the chain rule to be learned later for covering such situations.) I decline to get into a pedantic discussion of the distinction between f and f(z).
        $endgroup$
        – Henry Towsner
        Mar 21 at 18:14






      • 1




        $begingroup$
        I'm quite sure you use the chain rule to derive things that contain more than just $x$ in your calculus class, like $sqrtx^2+k$. You might say: that's ok if we treat $k$ as a constant and not as a variable. But then student might then ask: why am I not allowed to treat $x$ as a constant in the definition of $f$? (And a mathematician might add: what's the difference between a variable and a constant?). Apologies if my pedantry offends you.
        $endgroup$
        – Michael Bächtold
        Mar 21 at 18:49






      • 4




        $begingroup$
        @MichaelBächtold: In general, the difference between a variable and a constant is contextual and tricky to make precise, but for purposes of the chain rule in this case, x is a variable because we're taking the derivative with respect to it. I find that students don't usually have difficulty with this point (for instance, one could imagine students getting confused about the difference between the derivative of f(x)=c and f(x)=x, but that's not a particularly common issue), because it's a clean syntactic rule and it's backed up by the notion (x and z are conventionally variables, k isn't).
        $endgroup$
        – Henry Towsner
        Mar 21 at 19:28













      8












      8








      8





      $begingroup$

      f is not a function of (only) z - f here is a function of x as well as z. I think this explanation is intelligible to a calc 1 student, and gets at the heart of the matter.






      share|improve this answer









      $endgroup$



      f is not a function of (only) z - f here is a function of x as well as z. I think this explanation is intelligible to a calc 1 student, and gets at the heart of the matter.







      share|improve this answer












      share|improve this answer



      share|improve this answer










      answered Mar 21 at 16:30









      Henry TowsnerHenry Towsner

      7,1752350




      7,1752350







      • 2




        $begingroup$
        Hmm, so the student should reply: as soon as $f(x)$ contains parameters other than $x$ I am not allowed to apply the chain rule?
        $endgroup$
        – Michael Bächtold
        Mar 21 at 16:37











      • $begingroup$
        By the way: to my mind $f$ ist not a function of $z$ at all. Maybe you meant $f(z)$?
        $endgroup$
        – Michael Bächtold
        Mar 21 at 16:43






      • 1




        $begingroup$
        @MichaelBächtold: The student should know that if f(x) contains variables other than x then the chain rule doesn't apply. (This might be an opportunity to mention that there is a variant of the chain rule to be learned later for covering such situations.) I decline to get into a pedantic discussion of the distinction between f and f(z).
        $endgroup$
        – Henry Towsner
        Mar 21 at 18:14






      • 1




        $begingroup$
        I'm quite sure you use the chain rule to derive things that contain more than just $x$ in your calculus class, like $sqrtx^2+k$. You might say: that's ok if we treat $k$ as a constant and not as a variable. But then student might then ask: why am I not allowed to treat $x$ as a constant in the definition of $f$? (And a mathematician might add: what's the difference between a variable and a constant?). Apologies if my pedantry offends you.
        $endgroup$
        – Michael Bächtold
        Mar 21 at 18:49






      • 4




        $begingroup$
        @MichaelBächtold: In general, the difference between a variable and a constant is contextual and tricky to make precise, but for purposes of the chain rule in this case, x is a variable because we're taking the derivative with respect to it. I find that students don't usually have difficulty with this point (for instance, one could imagine students getting confused about the difference between the derivative of f(x)=c and f(x)=x, but that's not a particularly common issue), because it's a clean syntactic rule and it's backed up by the notion (x and z are conventionally variables, k isn't).
        $endgroup$
        – Henry Towsner
        Mar 21 at 19:28












      • 2




        $begingroup$
        Hmm, so the student should reply: as soon as $f(x)$ contains parameters other than $x$ I am not allowed to apply the chain rule?
        $endgroup$
        – Michael Bächtold
        Mar 21 at 16:37











      • $begingroup$
        By the way: to my mind $f$ ist not a function of $z$ at all. Maybe you meant $f(z)$?
        $endgroup$
        – Michael Bächtold
        Mar 21 at 16:43






      • 1




        $begingroup$
        @MichaelBächtold: The student should know that if f(x) contains variables other than x then the chain rule doesn't apply. (This might be an opportunity to mention that there is a variant of the chain rule to be learned later for covering such situations.) I decline to get into a pedantic discussion of the distinction between f and f(z).
        $endgroup$
        – Henry Towsner
        Mar 21 at 18:14






      • 1




        $begingroup$
        I'm quite sure you use the chain rule to derive things that contain more than just $x$ in your calculus class, like $sqrtx^2+k$. You might say: that's ok if we treat $k$ as a constant and not as a variable. But then student might then ask: why am I not allowed to treat $x$ as a constant in the definition of $f$? (And a mathematician might add: what's the difference between a variable and a constant?). Apologies if my pedantry offends you.
        $endgroup$
        – Michael Bächtold
        Mar 21 at 18:49






      • 4




        $begingroup$
        @MichaelBächtold: In general, the difference between a variable and a constant is contextual and tricky to make precise, but for purposes of the chain rule in this case, x is a variable because we're taking the derivative with respect to it. I find that students don't usually have difficulty with this point (for instance, one could imagine students getting confused about the difference between the derivative of f(x)=c and f(x)=x, but that's not a particularly common issue), because it's a clean syntactic rule and it's backed up by the notion (x and z are conventionally variables, k isn't).
        $endgroup$
        – Henry Towsner
        Mar 21 at 19:28







      2




      2




      $begingroup$
      Hmm, so the student should reply: as soon as $f(x)$ contains parameters other than $x$ I am not allowed to apply the chain rule?
      $endgroup$
      – Michael Bächtold
      Mar 21 at 16:37





      $begingroup$
      Hmm, so the student should reply: as soon as $f(x)$ contains parameters other than $x$ I am not allowed to apply the chain rule?
      $endgroup$
      – Michael Bächtold
      Mar 21 at 16:37













      $begingroup$
      By the way: to my mind $f$ ist not a function of $z$ at all. Maybe you meant $f(z)$?
      $endgroup$
      – Michael Bächtold
      Mar 21 at 16:43




      $begingroup$
      By the way: to my mind $f$ ist not a function of $z$ at all. Maybe you meant $f(z)$?
      $endgroup$
      – Michael Bächtold
      Mar 21 at 16:43




      1




      1




      $begingroup$
      @MichaelBächtold: The student should know that if f(x) contains variables other than x then the chain rule doesn't apply. (This might be an opportunity to mention that there is a variant of the chain rule to be learned later for covering such situations.) I decline to get into a pedantic discussion of the distinction between f and f(z).
      $endgroup$
      – Henry Towsner
      Mar 21 at 18:14




      $begingroup$
      @MichaelBächtold: The student should know that if f(x) contains variables other than x then the chain rule doesn't apply. (This might be an opportunity to mention that there is a variant of the chain rule to be learned later for covering such situations.) I decline to get into a pedantic discussion of the distinction between f and f(z).
      $endgroup$
      – Henry Towsner
      Mar 21 at 18:14




      1




      1




      $begingroup$
      I'm quite sure you use the chain rule to derive things that contain more than just $x$ in your calculus class, like $sqrtx^2+k$. You might say: that's ok if we treat $k$ as a constant and not as a variable. But then student might then ask: why am I not allowed to treat $x$ as a constant in the definition of $f$? (And a mathematician might add: what's the difference between a variable and a constant?). Apologies if my pedantry offends you.
      $endgroup$
      – Michael Bächtold
      Mar 21 at 18:49




      $begingroup$
      I'm quite sure you use the chain rule to derive things that contain more than just $x$ in your calculus class, like $sqrtx^2+k$. You might say: that's ok if we treat $k$ as a constant and not as a variable. But then student might then ask: why am I not allowed to treat $x$ as a constant in the definition of $f$? (And a mathematician might add: what's the difference between a variable and a constant?). Apologies if my pedantry offends you.
      $endgroup$
      – Michael Bächtold
      Mar 21 at 18:49




      4




      4




      $begingroup$
      @MichaelBächtold: In general, the difference between a variable and a constant is contextual and tricky to make precise, but for purposes of the chain rule in this case, x is a variable because we're taking the derivative with respect to it. I find that students don't usually have difficulty with this point (for instance, one could imagine students getting confused about the difference between the derivative of f(x)=c and f(x)=x, but that's not a particularly common issue), because it's a clean syntactic rule and it's backed up by the notion (x and z are conventionally variables, k isn't).
      $endgroup$
      – Henry Towsner
      Mar 21 at 19:28




      $begingroup$
      @MichaelBächtold: In general, the difference between a variable and a constant is contextual and tricky to make precise, but for purposes of the chain rule in this case, x is a variable because we're taking the derivative with respect to it. I find that students don't usually have difficulty with this point (for instance, one could imagine students getting confused about the difference between the derivative of f(x)=c and f(x)=x, but that's not a particularly common issue), because it's a clean syntactic rule and it's backed up by the notion (x and z are conventionally variables, k isn't).
      $endgroup$
      – Henry Towsner
      Mar 21 at 19:28











      4












      $begingroup$

      This is a VERY VERY typical problem. In fact, it's a problem even for $fracddx3^x$, much less your example.



      The way I try to deal with this is one of two ways.



      1. What has to happen first? To evaluate $3^5x+1$, you have to evaluate $5x+1$ first. So that is the inside function in the chain rule, just like in $sin(x^2)$ you have $x^2$ to evaluate first, so it is the inside function.


      2. You could rethink how we notate or talk about exponential functions. In particular, Excel has $e^x$ written as exp(x) (I think as an option). So one can ask what the "input" is here.


      However, on the plus side the student does seem to have the chain rule down; it's just the exponential notation that is causing trouble. So there is definitely hope here. And again, you should not be surprised at encountering this, so it is worth your time to come up with several possible responses for it in the long run. Good luck!






      share|improve this answer











      $endgroup$

















        4












        $begingroup$

        This is a VERY VERY typical problem. In fact, it's a problem even for $fracddx3^x$, much less your example.



        The way I try to deal with this is one of two ways.



        1. What has to happen first? To evaluate $3^5x+1$, you have to evaluate $5x+1$ first. So that is the inside function in the chain rule, just like in $sin(x^2)$ you have $x^2$ to evaluate first, so it is the inside function.


        2. You could rethink how we notate or talk about exponential functions. In particular, Excel has $e^x$ written as exp(x) (I think as an option). So one can ask what the "input" is here.


        However, on the plus side the student does seem to have the chain rule down; it's just the exponential notation that is causing trouble. So there is definitely hope here. And again, you should not be surprised at encountering this, so it is worth your time to come up with several possible responses for it in the long run. Good luck!






        share|improve this answer











        $endgroup$















          4












          4








          4





          $begingroup$

          This is a VERY VERY typical problem. In fact, it's a problem even for $fracddx3^x$, much less your example.



          The way I try to deal with this is one of two ways.



          1. What has to happen first? To evaluate $3^5x+1$, you have to evaluate $5x+1$ first. So that is the inside function in the chain rule, just like in $sin(x^2)$ you have $x^2$ to evaluate first, so it is the inside function.


          2. You could rethink how we notate or talk about exponential functions. In particular, Excel has $e^x$ written as exp(x) (I think as an option). So one can ask what the "input" is here.


          However, on the plus side the student does seem to have the chain rule down; it's just the exponential notation that is causing trouble. So there is definitely hope here. And again, you should not be surprised at encountering this, so it is worth your time to come up with several possible responses for it in the long run. Good luck!






          share|improve this answer











          $endgroup$



          This is a VERY VERY typical problem. In fact, it's a problem even for $fracddx3^x$, much less your example.



          The way I try to deal with this is one of two ways.



          1. What has to happen first? To evaluate $3^5x+1$, you have to evaluate $5x+1$ first. So that is the inside function in the chain rule, just like in $sin(x^2)$ you have $x^2$ to evaluate first, so it is the inside function.


          2. You could rethink how we notate or talk about exponential functions. In particular, Excel has $e^x$ written as exp(x) (I think as an option). So one can ask what the "input" is here.


          However, on the plus side the student does seem to have the chain rule down; it's just the exponential notation that is causing trouble. So there is definitely hope here. And again, you should not be surprised at encountering this, so it is worth your time to come up with several possible responses for it in the long run. Good luck!







          share|improve this answer














          share|improve this answer



          share|improve this answer








          edited Mar 23 at 1:09

























          answered Mar 21 at 19:37









          kcrismankcrisman

          3,633732




          3,633732





















              3












              $begingroup$

              This idea is fine, and you can use the multivariable chain rule to do it this way.



              Say we want to differentiate $h(x) = f(x)^g(x)$ with respect to $x$. Notice that we can write $h$ as the composite of $p: mathbbR to mathbbR^2$ defined by $p(t) = (f(t),g(t))$ with the function $E: mathbbR^2 to mathbbR$ defined by $E(u,v) = u^v$.



              By the multivariable chain rule,



              $$
              beginalign
              Dhbig|_x &= DEbig|_p(x) circ Dpbig|_x\
              &= left.beginbmatrix fracpartial Epartial u & fracpartial Epartial v endbmatrix right|_(u,v) = (f(x),g(x)) circ left.beginbmatrix fracpartial fpartial t \ fracpartial gpartial tendbmatrixright|_t = x\
              &= left.beginbmatrix vu^v-1 & ln(u) u^v endbmatrix right|_(u,v) = (f(x),g(x)) circ left.beginbmatrix f'(t) \ g'(t)endbmatrixright|_t = x\
              &= beginbmatrix g(x)(f(x))^g(x)-1 & ln(f(x)) (f(x))^g(x) endbmatrix circ beginbmatrix f'(x) \ g'(x)endbmatrix\
              &=f'(x)g(x)(f(x))^g(x)-1+g'(x)ln(x)f(x)^g(x)
              endalign
              $$



              Applying this to the problem in question, we see that $f'(x) =0$, so the first term disappears.



              So, in a sense, the student was trying to apply the multivariable chain rule (using the two variables $z$ and $x$), but didn't know how to do that yet. So you could tell them it is a good approach, but they will learn how to properly execute that approach in calc 3.






              share|improve this answer









              $endgroup$

















                3












                $begingroup$

                This idea is fine, and you can use the multivariable chain rule to do it this way.



                Say we want to differentiate $h(x) = f(x)^g(x)$ with respect to $x$. Notice that we can write $h$ as the composite of $p: mathbbR to mathbbR^2$ defined by $p(t) = (f(t),g(t))$ with the function $E: mathbbR^2 to mathbbR$ defined by $E(u,v) = u^v$.



                By the multivariable chain rule,



                $$
                beginalign
                Dhbig|_x &= DEbig|_p(x) circ Dpbig|_x\
                &= left.beginbmatrix fracpartial Epartial u & fracpartial Epartial v endbmatrix right|_(u,v) = (f(x),g(x)) circ left.beginbmatrix fracpartial fpartial t \ fracpartial gpartial tendbmatrixright|_t = x\
                &= left.beginbmatrix vu^v-1 & ln(u) u^v endbmatrix right|_(u,v) = (f(x),g(x)) circ left.beginbmatrix f'(t) \ g'(t)endbmatrixright|_t = x\
                &= beginbmatrix g(x)(f(x))^g(x)-1 & ln(f(x)) (f(x))^g(x) endbmatrix circ beginbmatrix f'(x) \ g'(x)endbmatrix\
                &=f'(x)g(x)(f(x))^g(x)-1+g'(x)ln(x)f(x)^g(x)
                endalign
                $$



                Applying this to the problem in question, we see that $f'(x) =0$, so the first term disappears.



                So, in a sense, the student was trying to apply the multivariable chain rule (using the two variables $z$ and $x$), but didn't know how to do that yet. So you could tell them it is a good approach, but they will learn how to properly execute that approach in calc 3.






                share|improve this answer









                $endgroup$















                  3












                  3








                  3





                  $begingroup$

                  This idea is fine, and you can use the multivariable chain rule to do it this way.



                  Say we want to differentiate $h(x) = f(x)^g(x)$ with respect to $x$. Notice that we can write $h$ as the composite of $p: mathbbR to mathbbR^2$ defined by $p(t) = (f(t),g(t))$ with the function $E: mathbbR^2 to mathbbR$ defined by $E(u,v) = u^v$.



                  By the multivariable chain rule,



                  $$
                  beginalign
                  Dhbig|_x &= DEbig|_p(x) circ Dpbig|_x\
                  &= left.beginbmatrix fracpartial Epartial u & fracpartial Epartial v endbmatrix right|_(u,v) = (f(x),g(x)) circ left.beginbmatrix fracpartial fpartial t \ fracpartial gpartial tendbmatrixright|_t = x\
                  &= left.beginbmatrix vu^v-1 & ln(u) u^v endbmatrix right|_(u,v) = (f(x),g(x)) circ left.beginbmatrix f'(t) \ g'(t)endbmatrixright|_t = x\
                  &= beginbmatrix g(x)(f(x))^g(x)-1 & ln(f(x)) (f(x))^g(x) endbmatrix circ beginbmatrix f'(x) \ g'(x)endbmatrix\
                  &=f'(x)g(x)(f(x))^g(x)-1+g'(x)ln(x)f(x)^g(x)
                  endalign
                  $$



                  Applying this to the problem in question, we see that $f'(x) =0$, so the first term disappears.



                  So, in a sense, the student was trying to apply the multivariable chain rule (using the two variables $z$ and $x$), but didn't know how to do that yet. So you could tell them it is a good approach, but they will learn how to properly execute that approach in calc 3.






                  share|improve this answer









                  $endgroup$



                  This idea is fine, and you can use the multivariable chain rule to do it this way.



                  Say we want to differentiate $h(x) = f(x)^g(x)$ with respect to $x$. Notice that we can write $h$ as the composite of $p: mathbbR to mathbbR^2$ defined by $p(t) = (f(t),g(t))$ with the function $E: mathbbR^2 to mathbbR$ defined by $E(u,v) = u^v$.



                  By the multivariable chain rule,



                  $$
                  beginalign
                  Dhbig|_x &= DEbig|_p(x) circ Dpbig|_x\
                  &= left.beginbmatrix fracpartial Epartial u & fracpartial Epartial v endbmatrix right|_(u,v) = (f(x),g(x)) circ left.beginbmatrix fracpartial fpartial t \ fracpartial gpartial tendbmatrixright|_t = x\
                  &= left.beginbmatrix vu^v-1 & ln(u) u^v endbmatrix right|_(u,v) = (f(x),g(x)) circ left.beginbmatrix f'(t) \ g'(t)endbmatrixright|_t = x\
                  &= beginbmatrix g(x)(f(x))^g(x)-1 & ln(f(x)) (f(x))^g(x) endbmatrix circ beginbmatrix f'(x) \ g'(x)endbmatrix\
                  &=f'(x)g(x)(f(x))^g(x)-1+g'(x)ln(x)f(x)^g(x)
                  endalign
                  $$



                  Applying this to the problem in question, we see that $f'(x) =0$, so the first term disappears.



                  So, in a sense, the student was trying to apply the multivariable chain rule (using the two variables $z$ and $x$), but didn't know how to do that yet. So you could tell them it is a good approach, but they will learn how to properly execute that approach in calc 3.







                  share|improve this answer












                  share|improve this answer



                  share|improve this answer










                  answered Mar 22 at 12:34









                  Steven GubkinSteven Gubkin

                  8,74712549




                  8,74712549





















                      1












                      $begingroup$

                      I had a similar problem with a student last week and could not succinctly explain why she could not select $e$ as the 'inner' function here: $$f(x) = e^8x +4$$



                      The best explanation I have seen thus far, Paul's Notes, explains it in this way:




                      Recall that the 'outside' function is the last operation that we would perform in an evaluation. In this case if we were to evaluate this function the last operation would be the exponential. Therefore, the outside function is the exponential function and the inside function is its exponent.







                      share|improve this answer









                      $endgroup$












                      • $begingroup$
                        Because 'e' is a constant? How would you explain it if instead of e, the base were '2'?
                        $endgroup$
                        – JoeTaxpayer
                        Mar 22 at 15:27










                      • $begingroup$
                        Yes this is because 'e' is a constant. Therefore, the explanation would also work when the base is '2'.
                        $endgroup$
                        – MT Kop
                        Mar 25 at 12:09










                      • $begingroup$
                        Agreed, that was what I was trying to suggest. The f(x) you shared has no ‘inner function’ .
                        $endgroup$
                        – JoeTaxpayer
                        Mar 25 at 12:21















                      1












                      $begingroup$

                      I had a similar problem with a student last week and could not succinctly explain why she could not select $e$ as the 'inner' function here: $$f(x) = e^8x +4$$



                      The best explanation I have seen thus far, Paul's Notes, explains it in this way:




                      Recall that the 'outside' function is the last operation that we would perform in an evaluation. In this case if we were to evaluate this function the last operation would be the exponential. Therefore, the outside function is the exponential function and the inside function is its exponent.







                      share|improve this answer









                      $endgroup$












                      • $begingroup$
                        Because 'e' is a constant? How would you explain it if instead of e, the base were '2'?
                        $endgroup$
                        – JoeTaxpayer
                        Mar 22 at 15:27










                      • $begingroup$
                        Yes this is because 'e' is a constant. Therefore, the explanation would also work when the base is '2'.
                        $endgroup$
                        – MT Kop
                        Mar 25 at 12:09










                      • $begingroup$
                        Agreed, that was what I was trying to suggest. The f(x) you shared has no ‘inner function’ .
                        $endgroup$
                        – JoeTaxpayer
                        Mar 25 at 12:21













                      1












                      1








                      1





                      $begingroup$

                      I had a similar problem with a student last week and could not succinctly explain why she could not select $e$ as the 'inner' function here: $$f(x) = e^8x +4$$



                      The best explanation I have seen thus far, Paul's Notes, explains it in this way:




                      Recall that the 'outside' function is the last operation that we would perform in an evaluation. In this case if we were to evaluate this function the last operation would be the exponential. Therefore, the outside function is the exponential function and the inside function is its exponent.







                      share|improve this answer









                      $endgroup$



                      I had a similar problem with a student last week and could not succinctly explain why she could not select $e$ as the 'inner' function here: $$f(x) = e^8x +4$$



                      The best explanation I have seen thus far, Paul's Notes, explains it in this way:




                      Recall that the 'outside' function is the last operation that we would perform in an evaluation. In this case if we were to evaluate this function the last operation would be the exponential. Therefore, the outside function is the exponential function and the inside function is its exponent.








                      share|improve this answer












                      share|improve this answer



                      share|improve this answer










                      answered Mar 22 at 10:52









                      MT KopMT Kop

                      192




                      192











                      • $begingroup$
                        Because 'e' is a constant? How would you explain it if instead of e, the base were '2'?
                        $endgroup$
                        – JoeTaxpayer
                        Mar 22 at 15:27










                      • $begingroup$
                        Yes this is because 'e' is a constant. Therefore, the explanation would also work when the base is '2'.
                        $endgroup$
                        – MT Kop
                        Mar 25 at 12:09










                      • $begingroup$
                        Agreed, that was what I was trying to suggest. The f(x) you shared has no ‘inner function’ .
                        $endgroup$
                        – JoeTaxpayer
                        Mar 25 at 12:21
















                      • $begingroup$
                        Because 'e' is a constant? How would you explain it if instead of e, the base were '2'?
                        $endgroup$
                        – JoeTaxpayer
                        Mar 22 at 15:27










                      • $begingroup$
                        Yes this is because 'e' is a constant. Therefore, the explanation would also work when the base is '2'.
                        $endgroup$
                        – MT Kop
                        Mar 25 at 12:09










                      • $begingroup$
                        Agreed, that was what I was trying to suggest. The f(x) you shared has no ‘inner function’ .
                        $endgroup$
                        – JoeTaxpayer
                        Mar 25 at 12:21















                      $begingroup$
                      Because 'e' is a constant? How would you explain it if instead of e, the base were '2'?
                      $endgroup$
                      – JoeTaxpayer
                      Mar 22 at 15:27




                      $begingroup$
                      Because 'e' is a constant? How would you explain it if instead of e, the base were '2'?
                      $endgroup$
                      – JoeTaxpayer
                      Mar 22 at 15:27












                      $begingroup$
                      Yes this is because 'e' is a constant. Therefore, the explanation would also work when the base is '2'.
                      $endgroup$
                      – MT Kop
                      Mar 25 at 12:09




                      $begingroup$
                      Yes this is because 'e' is a constant. Therefore, the explanation would also work when the base is '2'.
                      $endgroup$
                      – MT Kop
                      Mar 25 at 12:09












                      $begingroup$
                      Agreed, that was what I was trying to suggest. The f(x) you shared has no ‘inner function’ .
                      $endgroup$
                      – JoeTaxpayer
                      Mar 25 at 12:21




                      $begingroup$
                      Agreed, that was what I was trying to suggest. The f(x) you shared has no ‘inner function’ .
                      $endgroup$
                      – JoeTaxpayer
                      Mar 25 at 12:21











                      1












                      $begingroup$

                      The other answers have completely missed the mistake.
                      $
                      defrrmathbbR
                      $



                      Your student's error has nothing to do with exponentiatiation. Consider the following based on exactly the same error:




                      $colorredtextLet (???)$ $f(y) = x$ and $g(x) = 1$.



                      Then $1 = fracdxdx = (f∘g)'(x) = f'(g(x))·g'(x) = f'(1)·0 = 0$.




                      The error lies in the very first line! It is extremely obvious once you actually attempt to make it rigorous. Recall that to define a function you must provide a domain as well as a rule that specifies the output for each input in the domain. And of course the rule has to be meaningful in the context where you want to define the function. So see what you get:




                      $colorredtextLet (???)$ $f : rr→rr$ such that $f(y) = x$ for each $y∈rr$.



                      Let $g : rr→rr$ such that $g(x) = 1$ for every $x∈rr$.




                      The definition of $g$ is fine. The definition of $f$ is not fine! What on earth is $x$? The rule has to specify the output for each input $y∈rr$, so where did $x$ pop up from?



                      As explained above, the error has nothing to do with differentiation. Rather, it is in the illegal definition of the function!






                      share|improve this answer









                      $endgroup$












                      • $begingroup$
                        Furthermore, it is misleading to bring in Leibniz notation when the question is about ordinary functions. Even if we do, the answer given by Taemyr is simply wrong. Given any real/complex variables $x,y,z$, the proper chain rule asserts $fracdzdx = fracdzdy ·fracdydx$ if $fracdzdy$ and $fracdydx$ are both defined. If you cannot prove that the two derivative expressions are defined, then you're not allowed to even write down the so-called chain rule, because it doesn't apply!
                        $endgroup$
                        – user21820
                        Mar 24 at 8:17






                      • 1




                        $begingroup$
                        I upvoted your answer, because it contributes something mathematically to the discussion, even if it completely lacks any tact. I would be curious to know in what sense you think my answer "completely misses the mark": it points out the same error as yours while being slightly more generous to the student (assuming that they were actually defining a function of two variables, rather than just writing something completely meaningless as you suppose). It then shows how the mistake (under this assumption) can be corrected through correct use of the multivariable chain rule.
                        $endgroup$
                        – Steven Gubkin
                        Mar 24 at 14:24







                      • 1




                        $begingroup$
                        (cont.) without going into free/bound variables, $alpha$-conversion and variable capture. That's what Kevin did. Not that I find it wrong to go into this stuff, but I don't think it's so useful for calculus students coming from engineering. Instead, telling them that $3^5x+1$ is not a function of $3$ and hence I cannot write $d3^5x+1/d3$ seems closer to what they'll need. It's a pity that a hundred years since Frege, logicians have not been able to properly formalise the surrounding notions of variables, constants, and functions of things.
                        $endgroup$
                        – Michael Bächtold
                        Mar 24 at 20:45







                      • 2




                        $begingroup$
                        @MichaelBächtold: I'll respond in chat. This is getting way too long for a comment thread.
                        $endgroup$
                        – user21820
                        Mar 25 at 8:13






                      • 1




                        $begingroup$
                        For future readers, the discussion is continued in depth starting from here.
                        $endgroup$
                        – user21820
                        Mar 25 at 9:54















                      1












                      $begingroup$

                      The other answers have completely missed the mistake.
                      $
                      defrrmathbbR
                      $



                      Your student's error has nothing to do with exponentiatiation. Consider the following based on exactly the same error:




                      $colorredtextLet (???)$ $f(y) = x$ and $g(x) = 1$.



                      Then $1 = fracdxdx = (f∘g)'(x) = f'(g(x))·g'(x) = f'(1)·0 = 0$.




                      The error lies in the very first line! It is extremely obvious once you actually attempt to make it rigorous. Recall that to define a function you must provide a domain as well as a rule that specifies the output for each input in the domain. And of course the rule has to be meaningful in the context where you want to define the function. So see what you get:




                      $colorredtextLet (???)$ $f : rr→rr$ such that $f(y) = x$ for each $y∈rr$.



                      Let $g : rr→rr$ such that $g(x) = 1$ for every $x∈rr$.




                      The definition of $g$ is fine. The definition of $f$ is not fine! What on earth is $x$? The rule has to specify the output for each input $y∈rr$, so where did $x$ pop up from?



                      As explained above, the error has nothing to do with differentiation. Rather, it is in the illegal definition of the function!






                      share|improve this answer









                      $endgroup$












                      • $begingroup$
                        Furthermore, it is misleading to bring in Leibniz notation when the question is about ordinary functions. Even if we do, the answer given by Taemyr is simply wrong. Given any real/complex variables $x,y,z$, the proper chain rule asserts $fracdzdx = fracdzdy ·fracdydx$ if $fracdzdy$ and $fracdydx$ are both defined. If you cannot prove that the two derivative expressions are defined, then you're not allowed to even write down the so-called chain rule, because it doesn't apply!
                        $endgroup$
                        – user21820
                        Mar 24 at 8:17






                      • 1




                        $begingroup$
                        I upvoted your answer, because it contributes something mathematically to the discussion, even if it completely lacks any tact. I would be curious to know in what sense you think my answer "completely misses the mark": it points out the same error as yours while being slightly more generous to the student (assuming that they were actually defining a function of two variables, rather than just writing something completely meaningless as you suppose). It then shows how the mistake (under this assumption) can be corrected through correct use of the multivariable chain rule.
                        $endgroup$
                        – Steven Gubkin
                        Mar 24 at 14:24







                      • 1




                        $begingroup$
                        (cont.) without going into free/bound variables, $alpha$-conversion and variable capture. That's what Kevin did. Not that I find it wrong to go into this stuff, but I don't think it's so useful for calculus students coming from engineering. Instead, telling them that $3^5x+1$ is not a function of $3$ and hence I cannot write $d3^5x+1/d3$ seems closer to what they'll need. It's a pity that a hundred years since Frege, logicians have not been able to properly formalise the surrounding notions of variables, constants, and functions of things.
                        $endgroup$
                        – Michael Bächtold
                        Mar 24 at 20:45







                      • 2




                        $begingroup$
                        @MichaelBächtold: I'll respond in chat. This is getting way too long for a comment thread.
                        $endgroup$
                        – user21820
                        Mar 25 at 8:13






                      • 1




                        $begingroup$
                        For future readers, the discussion is continued in depth starting from here.
                        $endgroup$
                        – user21820
                        Mar 25 at 9:54













                      1












                      1








                      1





                      $begingroup$

                      The other answers have completely missed the mistake.
                      $
                      defrrmathbbR
                      $



                      Your student's error has nothing to do with exponentiatiation. Consider the following based on exactly the same error:




                      $colorredtextLet (???)$ $f(y) = x$ and $g(x) = 1$.



                      Then $1 = fracdxdx = (f∘g)'(x) = f'(g(x))·g'(x) = f'(1)·0 = 0$.




                      The error lies in the very first line! It is extremely obvious once you actually attempt to make it rigorous. Recall that to define a function you must provide a domain as well as a rule that specifies the output for each input in the domain. And of course the rule has to be meaningful in the context where you want to define the function. So see what you get:




                      $colorredtextLet (???)$ $f : rr→rr$ such that $f(y) = x$ for each $y∈rr$.



                      Let $g : rr→rr$ such that $g(x) = 1$ for every $x∈rr$.




                      The definition of $g$ is fine. The definition of $f$ is not fine! What on earth is $x$? The rule has to specify the output for each input $y∈rr$, so where did $x$ pop up from?



                      As explained above, the error has nothing to do with differentiation. Rather, it is in the illegal definition of the function!






                      share|improve this answer









                      $endgroup$



                      The other answers have completely missed the mistake.
                      $
                      defrrmathbbR
                      $



                      Your student's error has nothing to do with exponentiatiation. Consider the following based on exactly the same error:




                      $colorredtextLet (???)$ $f(y) = x$ and $g(x) = 1$.



                      Then $1 = fracdxdx = (f∘g)'(x) = f'(g(x))·g'(x) = f'(1)·0 = 0$.




                      The error lies in the very first line! It is extremely obvious once you actually attempt to make it rigorous. Recall that to define a function you must provide a domain as well as a rule that specifies the output for each input in the domain. And of course the rule has to be meaningful in the context where you want to define the function. So see what you get:




                      $colorredtextLet (???)$ $f : rr→rr$ such that $f(y) = x$ for each $y∈rr$.



                      Let $g : rr→rr$ such that $g(x) = 1$ for every $x∈rr$.




                      The definition of $g$ is fine. The definition of $f$ is not fine! What on earth is $x$? The rule has to specify the output for each input $y∈rr$, so where did $x$ pop up from?



                      As explained above, the error has nothing to do with differentiation. Rather, it is in the illegal definition of the function!







                      share|improve this answer












                      share|improve this answer



                      share|improve this answer










                      answered Mar 24 at 8:06









                      user21820user21820

                      1,174717




                      1,174717











                      • $begingroup$
                        Furthermore, it is misleading to bring in Leibniz notation when the question is about ordinary functions. Even if we do, the answer given by Taemyr is simply wrong. Given any real/complex variables $x,y,z$, the proper chain rule asserts $fracdzdx = fracdzdy ·fracdydx$ if $fracdzdy$ and $fracdydx$ are both defined. If you cannot prove that the two derivative expressions are defined, then you're not allowed to even write down the so-called chain rule, because it doesn't apply!
                        $endgroup$
                        – user21820
                        Mar 24 at 8:17






                      • 1




                        $begingroup$
                        I upvoted your answer, because it contributes something mathematically to the discussion, even if it completely lacks any tact. I would be curious to know in what sense you think my answer "completely misses the mark": it points out the same error as yours while being slightly more generous to the student (assuming that they were actually defining a function of two variables, rather than just writing something completely meaningless as you suppose). It then shows how the mistake (under this assumption) can be corrected through correct use of the multivariable chain rule.
                        $endgroup$
                        – Steven Gubkin
                        Mar 24 at 14:24







                      • 1




                        $begingroup$
                        (cont.) without going into free/bound variables, $alpha$-conversion and variable capture. That's what Kevin did. Not that I find it wrong to go into this stuff, but I don't think it's so useful for calculus students coming from engineering. Instead, telling them that $3^5x+1$ is not a function of $3$ and hence I cannot write $d3^5x+1/d3$ seems closer to what they'll need. It's a pity that a hundred years since Frege, logicians have not been able to properly formalise the surrounding notions of variables, constants, and functions of things.
                        $endgroup$
                        – Michael Bächtold
                        Mar 24 at 20:45







                      • 2




                        $begingroup$
                        @MichaelBächtold: I'll respond in chat. This is getting way too long for a comment thread.
                        $endgroup$
                        – user21820
                        Mar 25 at 8:13






                      • 1




                        $begingroup$
                        For future readers, the discussion is continued in depth starting from here.
                        $endgroup$
                        – user21820
                        Mar 25 at 9:54
















                      • $begingroup$
                        Furthermore, it is misleading to bring in Leibniz notation when the question is about ordinary functions. Even if we do, the answer given by Taemyr is simply wrong. Given any real/complex variables $x,y,z$, the proper chain rule asserts $fracdzdx = fracdzdy ·fracdydx$ if $fracdzdy$ and $fracdydx$ are both defined. If you cannot prove that the two derivative expressions are defined, then you're not allowed to even write down the so-called chain rule, because it doesn't apply!
                        $endgroup$
                        – user21820
                        Mar 24 at 8:17






                      • 1




                        $begingroup$
                        I upvoted your answer, because it contributes something mathematically to the discussion, even if it completely lacks any tact. I would be curious to know in what sense you think my answer "completely misses the mark": it points out the same error as yours while being slightly more generous to the student (assuming that they were actually defining a function of two variables, rather than just writing something completely meaningless as you suppose). It then shows how the mistake (under this assumption) can be corrected through correct use of the multivariable chain rule.
                        $endgroup$
                        – Steven Gubkin
                        Mar 24 at 14:24







                      • 1




                        $begingroup$
                        (cont.) without going into free/bound variables, $alpha$-conversion and variable capture. That's what Kevin did. Not that I find it wrong to go into this stuff, but I don't think it's so useful for calculus students coming from engineering. Instead, telling them that $3^5x+1$ is not a function of $3$ and hence I cannot write $d3^5x+1/d3$ seems closer to what they'll need. It's a pity that a hundred years since Frege, logicians have not been able to properly formalise the surrounding notions of variables, constants, and functions of things.
                        $endgroup$
                        – Michael Bächtold
                        Mar 24 at 20:45







                      • 2




                        $begingroup$
                        @MichaelBächtold: I'll respond in chat. This is getting way too long for a comment thread.
                        $endgroup$
                        – user21820
                        Mar 25 at 8:13






                      • 1




                        $begingroup$
                        For future readers, the discussion is continued in depth starting from here.
                        $endgroup$
                        – user21820
                        Mar 25 at 9:54















                      $begingroup$
                      Furthermore, it is misleading to bring in Leibniz notation when the question is about ordinary functions. Even if we do, the answer given by Taemyr is simply wrong. Given any real/complex variables $x,y,z$, the proper chain rule asserts $fracdzdx = fracdzdy ·fracdydx$ if $fracdzdy$ and $fracdydx$ are both defined. If you cannot prove that the two derivative expressions are defined, then you're not allowed to even write down the so-called chain rule, because it doesn't apply!
                      $endgroup$
                      – user21820
                      Mar 24 at 8:17




                      $begingroup$
                      Furthermore, it is misleading to bring in Leibniz notation when the question is about ordinary functions. Even if we do, the answer given by Taemyr is simply wrong. Given any real/complex variables $x,y,z$, the proper chain rule asserts $fracdzdx = fracdzdy ·fracdydx$ if $fracdzdy$ and $fracdydx$ are both defined. If you cannot prove that the two derivative expressions are defined, then you're not allowed to even write down the so-called chain rule, because it doesn't apply!
                      $endgroup$
                      – user21820
                      Mar 24 at 8:17




                      1




                      1




                      $begingroup$
                      I upvoted your answer, because it contributes something mathematically to the discussion, even if it completely lacks any tact. I would be curious to know in what sense you think my answer "completely misses the mark": it points out the same error as yours while being slightly more generous to the student (assuming that they were actually defining a function of two variables, rather than just writing something completely meaningless as you suppose). It then shows how the mistake (under this assumption) can be corrected through correct use of the multivariable chain rule.
                      $endgroup$
                      – Steven Gubkin
                      Mar 24 at 14:24





                      $begingroup$
                      I upvoted your answer, because it contributes something mathematically to the discussion, even if it completely lacks any tact. I would be curious to know in what sense you think my answer "completely misses the mark": it points out the same error as yours while being slightly more generous to the student (assuming that they were actually defining a function of two variables, rather than just writing something completely meaningless as you suppose). It then shows how the mistake (under this assumption) can be corrected through correct use of the multivariable chain rule.
                      $endgroup$
                      – Steven Gubkin
                      Mar 24 at 14:24





                      1




                      1




                      $begingroup$
                      (cont.) without going into free/bound variables, $alpha$-conversion and variable capture. That's what Kevin did. Not that I find it wrong to go into this stuff, but I don't think it's so useful for calculus students coming from engineering. Instead, telling them that $3^5x+1$ is not a function of $3$ and hence I cannot write $d3^5x+1/d3$ seems closer to what they'll need. It's a pity that a hundred years since Frege, logicians have not been able to properly formalise the surrounding notions of variables, constants, and functions of things.
                      $endgroup$
                      – Michael Bächtold
                      Mar 24 at 20:45





                      $begingroup$
                      (cont.) without going into free/bound variables, $alpha$-conversion and variable capture. That's what Kevin did. Not that I find it wrong to go into this stuff, but I don't think it's so useful for calculus students coming from engineering. Instead, telling them that $3^5x+1$ is not a function of $3$ and hence I cannot write $d3^5x+1/d3$ seems closer to what they'll need. It's a pity that a hundred years since Frege, logicians have not been able to properly formalise the surrounding notions of variables, constants, and functions of things.
                      $endgroup$
                      – Michael Bächtold
                      Mar 24 at 20:45





                      2




                      2




                      $begingroup$
                      @MichaelBächtold: I'll respond in chat. This is getting way too long for a comment thread.
                      $endgroup$
                      – user21820
                      Mar 25 at 8:13




                      $begingroup$
                      @MichaelBächtold: I'll respond in chat. This is getting way too long for a comment thread.
                      $endgroup$
                      – user21820
                      Mar 25 at 8:13




                      1




                      1




                      $begingroup$
                      For future readers, the discussion is continued in depth starting from here.
                      $endgroup$
                      – user21820
                      Mar 25 at 9:54




                      $begingroup$
                      For future readers, the discussion is continued in depth starting from here.
                      $endgroup$
                      – user21820
                      Mar 25 at 9:54











                      1












                      $begingroup$

                      Some users have expressed doubt at the validity of the accepted answer, so let me make it rigorous. To do that, we first need to make sense of the notation $fracdydx$. (Which is not a trivial task.)



                      We start by changing perspective: instead of thinking of variables $x$ and $y$ as numbers, we think of them as smooth real valued functions on some manifold $M$. So $xcolon Mto mathbbR$ and $ycolon Mto mathbbR$. You might rightly ask: why would we do that and which manifold $M$ are you talking about? The answer to the first question is: because I'd like to talk about the differentials $dy$, $dx$ and say everyday stuff like "$y$ is a constant" or "$y$ is a function of $x$". All of that is impossible inside first order logic + ZFC if we simply interpret $x$ and $y$ as elements of $mathbbR$. Concerning the second question: think of $M$ as the physical state space underlying the problem we are trying to model and $x$ and $y$ as observables. If that sounds too unfamiliar: it's similar to how people in probability theory assume an underlying space of outcomes $Omega$, in order to talk about random variables. (Which are the things we really care about and historically came before $Omega$, just like $dx,dy$ historically came before manifolds, but I'm drifting of.)



                      So, having fixed the background manifold $M$, whenever you hear me say variable, what I mean is a thing of type $Mto mathbbR$.




                      Definition. Given two variables $x$ and $y$, call $y$ a function of $x$ if $dx$ is not zero almost everywhere and there exists a variable $qcolon M to mathbbR$ such that
                      $$
                      dy = q cdot dx.
                      $$




                      Intuitively, the equation $
                      dy = q cdot dx
                      $
                      says that the change of $y$ is determined by the change of $x$, i.e. that $y$ depends on $x$.



                      It's not hard to show that $q$ is uniquely determined by $x$ and $y$, hence we decide to denote it with $fracdydx$ and call it the derivative of $y$ wrt. $x$. It was originally called the differential coefficient, cause that's what it is.



                      According to this definition, $3^5x+1$ is a function of $x$ (assuming $x$ is a true variabel, i.e. $dxneq 0$), but it's also a function of $5x+1$. On the other hand, $3^5x+1$ is not a function of $3$ since $d3=0$ (what we call a constant). In particular $fracd3^5x+1d3$ is undefined, as user21820 has been pointing out emphatically.



                      We can now state the chain rule in Leibniz form




                      Theorem. If $z$ is a function of $y$ and $y$ is a function of $x$, then $z$ is also a function $x$ and their differential coefficients satisfy
                      $$
                      fracdzdx=fracdzdycdot fracdydx
                      $$




                      The proof of this is trivial.



                      From this perspective, what the student in my question was trying to do, was to let $z=3^5x+1$ and $y=3$. But the theorem does not apply, since $3^5x+1$ is not a function $3$. The same point of view can be used in the example discussed here, where a student attempted to differentiate $x^x$ by taking $x$ as inner function. Although that's allowed it just leads to
                      $$
                      fracdx^xdx=fracdx^xdxcdot fracdxdx
                      $$

                      which is of not much use.



                      This is not to say that I don't appreciate the other answers (also users 21820). Taemyr's is just one of the three perspectives that haven been proposed. It might seem like it needs a lot of background to make it rigorous. But consider that mathematicians understood this stuff for at least 200 years without requiring manifolds to formalize it. And consider that the other approaches also require quite some background to make them rigorous (like quantifiers, variable bindings, the idea of dummy/bound variables etc. or derivatives of functions of multiple variables). Each has it advantages and disadvantages and none seems more right than the others, methinks.






                      share|improve this answer









                      $endgroup$












                      • $begingroup$
                        It seems you are finally agreeing with me that the accepted answer is not completely correct, because it literally wrote an equation involving what your answer states to be ill-defined. That from the beginning was my objection to it (see my first comment). I did not say that anything else was amiss with that answer, except that it fails to pinpoint the original student's error (regarding functions, not differentials).
                        $endgroup$
                        – user21820
                        Mar 25 at 18:34











                      • $begingroup$
                        @user21820 You have a strange sense of humor. But thanks for making me laugh. Unfortunately you also made me give up my last hope of leading an honest conversation with you.
                        $endgroup$
                        – Michael Bächtold
                        Mar 26 at 8:33










                      • $begingroup$
                        It's your choice, but you've been repeatedly misinterpreting whatever I say, and then blame me for it instead of considering that perhaps you are the one who is wrong. Your last comment insinuates that I am dishonest. That's false.
                        $endgroup$
                        – user21820
                        Mar 26 at 9:05















                      1












                      $begingroup$

                      Some users have expressed doubt at the validity of the accepted answer, so let me make it rigorous. To do that, we first need to make sense of the notation $fracdydx$. (Which is not a trivial task.)



                      We start by changing perspective: instead of thinking of variables $x$ and $y$ as numbers, we think of them as smooth real valued functions on some manifold $M$. So $xcolon Mto mathbbR$ and $ycolon Mto mathbbR$. You might rightly ask: why would we do that and which manifold $M$ are you talking about? The answer to the first question is: because I'd like to talk about the differentials $dy$, $dx$ and say everyday stuff like "$y$ is a constant" or "$y$ is a function of $x$". All of that is impossible inside first order logic + ZFC if we simply interpret $x$ and $y$ as elements of $mathbbR$. Concerning the second question: think of $M$ as the physical state space underlying the problem we are trying to model and $x$ and $y$ as observables. If that sounds too unfamiliar: it's similar to how people in probability theory assume an underlying space of outcomes $Omega$, in order to talk about random variables. (Which are the things we really care about and historically came before $Omega$, just like $dx,dy$ historically came before manifolds, but I'm drifting of.)



                      So, having fixed the background manifold $M$, whenever you hear me say variable, what I mean is a thing of type $Mto mathbbR$.




                      Definition. Given two variables $x$ and $y$, call $y$ a function of $x$ if $dx$ is not zero almost everywhere and there exists a variable $qcolon M to mathbbR$ such that
                      $$
                      dy = q cdot dx.
                      $$




                      Intuitively, the equation $
                      dy = q cdot dx
                      $
                      says that the change of $y$ is determined by the change of $x$, i.e. that $y$ depends on $x$.



                      It's not hard to show that $q$ is uniquely determined by $x$ and $y$, hence we decide to denote it with $fracdydx$ and call it the derivative of $y$ wrt. $x$. It was originally called the differential coefficient, cause that's what it is.



                      According to this definition, $3^5x+1$ is a function of $x$ (assuming $x$ is a true variabel, i.e. $dxneq 0$), but it's also a function of $5x+1$. On the other hand, $3^5x+1$ is not a function of $3$ since $d3=0$ (what we call a constant). In particular $fracd3^5x+1d3$ is undefined, as user21820 has been pointing out emphatically.



                      We can now state the chain rule in Leibniz form




                      Theorem. If $z$ is a function of $y$ and $y$ is a function of $x$, then $z$ is also a function $x$ and their differential coefficients satisfy
                      $$
                      fracdzdx=fracdzdycdot fracdydx
                      $$




                      The proof of this is trivial.



                      From this perspective, what the student in my question was trying to do, was to let $z=3^5x+1$ and $y=3$. But the theorem does not apply, since $3^5x+1$ is not a function $3$. The same point of view can be used in the example discussed here, where a student attempted to differentiate $x^x$ by taking $x$ as inner function. Although that's allowed it just leads to
                      $$
                      fracdx^xdx=fracdx^xdxcdot fracdxdx
                      $$

                      which is of not much use.



                      This is not to say that I don't appreciate the other answers (also users 21820). Taemyr's is just one of the three perspectives that haven been proposed. It might seem like it needs a lot of background to make it rigorous. But consider that mathematicians understood this stuff for at least 200 years without requiring manifolds to formalize it. And consider that the other approaches also require quite some background to make them rigorous (like quantifiers, variable bindings, the idea of dummy/bound variables etc. or derivatives of functions of multiple variables). Each has it advantages and disadvantages and none seems more right than the others, methinks.






                      share|improve this answer









                      $endgroup$












                      • $begingroup$
                        It seems you are finally agreeing with me that the accepted answer is not completely correct, because it literally wrote an equation involving what your answer states to be ill-defined. That from the beginning was my objection to it (see my first comment). I did not say that anything else was amiss with that answer, except that it fails to pinpoint the original student's error (regarding functions, not differentials).
                        $endgroup$
                        – user21820
                        Mar 25 at 18:34











                      • $begingroup$
                        @user21820 You have a strange sense of humor. But thanks for making me laugh. Unfortunately you also made me give up my last hope of leading an honest conversation with you.
                        $endgroup$
                        – Michael Bächtold
                        Mar 26 at 8:33










                      • $begingroup$
                        It's your choice, but you've been repeatedly misinterpreting whatever I say, and then blame me for it instead of considering that perhaps you are the one who is wrong. Your last comment insinuates that I am dishonest. That's false.
                        $endgroup$
                        – user21820
                        Mar 26 at 9:05













                      1












                      1








                      1





                      $begingroup$

                      Some users have expressed doubt at the validity of the accepted answer, so let me make it rigorous. To do that, we first need to make sense of the notation $fracdydx$. (Which is not a trivial task.)



                      We start by changing perspective: instead of thinking of variables $x$ and $y$ as numbers, we think of them as smooth real valued functions on some manifold $M$. So $xcolon Mto mathbbR$ and $ycolon Mto mathbbR$. You might rightly ask: why would we do that and which manifold $M$ are you talking about? The answer to the first question is: because I'd like to talk about the differentials $dy$, $dx$ and say everyday stuff like "$y$ is a constant" or "$y$ is a function of $x$". All of that is impossible inside first order logic + ZFC if we simply interpret $x$ and $y$ as elements of $mathbbR$. Concerning the second question: think of $M$ as the physical state space underlying the problem we are trying to model and $x$ and $y$ as observables. If that sounds too unfamiliar: it's similar to how people in probability theory assume an underlying space of outcomes $Omega$, in order to talk about random variables. (Which are the things we really care about and historically came before $Omega$, just like $dx,dy$ historically came before manifolds, but I'm drifting of.)



                      So, having fixed the background manifold $M$, whenever you hear me say variable, what I mean is a thing of type $Mto mathbbR$.




                      Definition. Given two variables $x$ and $y$, call $y$ a function of $x$ if $dx$ is not zero almost everywhere and there exists a variable $qcolon M to mathbbR$ such that
                      $$
                      dy = q cdot dx.
                      $$




                      Intuitively, the equation $
                      dy = q cdot dx
                      $
                      says that the change of $y$ is determined by the change of $x$, i.e. that $y$ depends on $x$.



                      It's not hard to show that $q$ is uniquely determined by $x$ and $y$, hence we decide to denote it with $fracdydx$ and call it the derivative of $y$ wrt. $x$. It was originally called the differential coefficient, cause that's what it is.



                      According to this definition, $3^5x+1$ is a function of $x$ (assuming $x$ is a true variabel, i.e. $dxneq 0$), but it's also a function of $5x+1$. On the other hand, $3^5x+1$ is not a function of $3$ since $d3=0$ (what we call a constant). In particular $fracd3^5x+1d3$ is undefined, as user21820 has been pointing out emphatically.



                      We can now state the chain rule in Leibniz form




                      Theorem. If $z$ is a function of $y$ and $y$ is a function of $x$, then $z$ is also a function $x$ and their differential coefficients satisfy
                      $$
                      fracdzdx=fracdzdycdot fracdydx
                      $$




                      The proof of this is trivial.



                      From this perspective, what the student in my question was trying to do, was to let $z=3^5x+1$ and $y=3$. But the theorem does not apply, since $3^5x+1$ is not a function $3$. The same point of view can be used in the example discussed here, where a student attempted to differentiate $x^x$ by taking $x$ as inner function. Although that's allowed it just leads to
                      $$
                      fracdx^xdx=fracdx^xdxcdot fracdxdx
                      $$

                      which is of not much use.



                      This is not to say that I don't appreciate the other answers (also users 21820). Taemyr's is just one of the three perspectives that haven been proposed. It might seem like it needs a lot of background to make it rigorous. But consider that mathematicians understood this stuff for at least 200 years without requiring manifolds to formalize it. And consider that the other approaches also require quite some background to make them rigorous (like quantifiers, variable bindings, the idea of dummy/bound variables etc. or derivatives of functions of multiple variables). Each has it advantages and disadvantages and none seems more right than the others, methinks.






                      share|improve this answer









                      $endgroup$



                      Some users have expressed doubt at the validity of the accepted answer, so let me make it rigorous. To do that, we first need to make sense of the notation $fracdydx$. (Which is not a trivial task.)



                      We start by changing perspective: instead of thinking of variables $x$ and $y$ as numbers, we think of them as smooth real valued functions on some manifold $M$. So $xcolon Mto mathbbR$ and $ycolon Mto mathbbR$. You might rightly ask: why would we do that and which manifold $M$ are you talking about? The answer to the first question is: because I'd like to talk about the differentials $dy$, $dx$ and say everyday stuff like "$y$ is a constant" or "$y$ is a function of $x$". All of that is impossible inside first order logic + ZFC if we simply interpret $x$ and $y$ as elements of $mathbbR$. Concerning the second question: think of $M$ as the physical state space underlying the problem we are trying to model and $x$ and $y$ as observables. If that sounds too unfamiliar: it's similar to how people in probability theory assume an underlying space of outcomes $Omega$, in order to talk about random variables. (Which are the things we really care about and historically came before $Omega$, just like $dx,dy$ historically came before manifolds, but I'm drifting of.)



                      So, having fixed the background manifold $M$, whenever you hear me say variable, what I mean is a thing of type $Mto mathbbR$.




                      Definition. Given two variables $x$ and $y$, call $y$ a function of $x$ if $dx$ is not zero almost everywhere and there exists a variable $qcolon M to mathbbR$ such that
                      $$
                      dy = q cdot dx.
                      $$




                      Intuitively, the equation $
                      dy = q cdot dx
                      $
                      says that the change of $y$ is determined by the change of $x$, i.e. that $y$ depends on $x$.



                      It's not hard to show that $q$ is uniquely determined by $x$ and $y$, hence we decide to denote it with $fracdydx$ and call it the derivative of $y$ wrt. $x$. It was originally called the differential coefficient, cause that's what it is.



                      According to this definition, $3^5x+1$ is a function of $x$ (assuming $x$ is a true variabel, i.e. $dxneq 0$), but it's also a function of $5x+1$. On the other hand, $3^5x+1$ is not a function of $3$ since $d3=0$ (what we call a constant). In particular $fracd3^5x+1d3$ is undefined, as user21820 has been pointing out emphatically.



                      We can now state the chain rule in Leibniz form




                      Theorem. If $z$ is a function of $y$ and $y$ is a function of $x$, then $z$ is also a function $x$ and their differential coefficients satisfy
                      $$
                      fracdzdx=fracdzdycdot fracdydx
                      $$




                      The proof of this is trivial.



                      From this perspective, what the student in my question was trying to do, was to let $z=3^5x+1$ and $y=3$. But the theorem does not apply, since $3^5x+1$ is not a function $3$. The same point of view can be used in the example discussed here, where a student attempted to differentiate $x^x$ by taking $x$ as inner function. Although that's allowed it just leads to
                      $$
                      fracdx^xdx=fracdx^xdxcdot fracdxdx
                      $$

                      which is of not much use.



                      This is not to say that I don't appreciate the other answers (also users 21820). Taemyr's is just one of the three perspectives that haven been proposed. It might seem like it needs a lot of background to make it rigorous. But consider that mathematicians understood this stuff for at least 200 years without requiring manifolds to formalize it. And consider that the other approaches also require quite some background to make them rigorous (like quantifiers, variable bindings, the idea of dummy/bound variables etc. or derivatives of functions of multiple variables). Each has it advantages and disadvantages and none seems more right than the others, methinks.







                      share|improve this answer












                      share|improve this answer



                      share|improve this answer










                      answered Mar 25 at 15:41









                      Michael BächtoldMichael Bächtold

                      646515




                      646515











                      • $begingroup$
                        It seems you are finally agreeing with me that the accepted answer is not completely correct, because it literally wrote an equation involving what your answer states to be ill-defined. That from the beginning was my objection to it (see my first comment). I did not say that anything else was amiss with that answer, except that it fails to pinpoint the original student's error (regarding functions, not differentials).
                        $endgroup$
                        – user21820
                        Mar 25 at 18:34











                      • $begingroup$
                        @user21820 You have a strange sense of humor. But thanks for making me laugh. Unfortunately you also made me give up my last hope of leading an honest conversation with you.
                        $endgroup$
                        – Michael Bächtold
                        Mar 26 at 8:33










                      • $begingroup$
                        It's your choice, but you've been repeatedly misinterpreting whatever I say, and then blame me for it instead of considering that perhaps you are the one who is wrong. Your last comment insinuates that I am dishonest. That's false.
                        $endgroup$
                        – user21820
                        Mar 26 at 9:05
















                      • $begingroup$
                        It seems you are finally agreeing with me that the accepted answer is not completely correct, because it literally wrote an equation involving what your answer states to be ill-defined. That from the beginning was my objection to it (see my first comment). I did not say that anything else was amiss with that answer, except that it fails to pinpoint the original student's error (regarding functions, not differentials).
                        $endgroup$
                        – user21820
                        Mar 25 at 18:34











                      • $begingroup$
                        @user21820 You have a strange sense of humor. But thanks for making me laugh. Unfortunately you also made me give up my last hope of leading an honest conversation with you.
                        $endgroup$
                        – Michael Bächtold
                        Mar 26 at 8:33










                      • $begingroup$
                        It's your choice, but you've been repeatedly misinterpreting whatever I say, and then blame me for it instead of considering that perhaps you are the one who is wrong. Your last comment insinuates that I am dishonest. That's false.
                        $endgroup$
                        – user21820
                        Mar 26 at 9:05















                      $begingroup$
                      It seems you are finally agreeing with me that the accepted answer is not completely correct, because it literally wrote an equation involving what your answer states to be ill-defined. That from the beginning was my objection to it (see my first comment). I did not say that anything else was amiss with that answer, except that it fails to pinpoint the original student's error (regarding functions, not differentials).
                      $endgroup$
                      – user21820
                      Mar 25 at 18:34





                      $begingroup$
                      It seems you are finally agreeing with me that the accepted answer is not completely correct, because it literally wrote an equation involving what your answer states to be ill-defined. That from the beginning was my objection to it (see my first comment). I did not say that anything else was amiss with that answer, except that it fails to pinpoint the original student's error (regarding functions, not differentials).
                      $endgroup$
                      – user21820
                      Mar 25 at 18:34













                      $begingroup$
                      @user21820 You have a strange sense of humor. But thanks for making me laugh. Unfortunately you also made me give up my last hope of leading an honest conversation with you.
                      $endgroup$
                      – Michael Bächtold
                      Mar 26 at 8:33




                      $begingroup$
                      @user21820 You have a strange sense of humor. But thanks for making me laugh. Unfortunately you also made me give up my last hope of leading an honest conversation with you.
                      $endgroup$
                      – Michael Bächtold
                      Mar 26 at 8:33












                      $begingroup$
                      It's your choice, but you've been repeatedly misinterpreting whatever I say, and then blame me for it instead of considering that perhaps you are the one who is wrong. Your last comment insinuates that I am dishonest. That's false.
                      $endgroup$
                      – user21820
                      Mar 26 at 9:05




                      $begingroup$
                      It's your choice, but you've been repeatedly misinterpreting whatever I say, and then blame me for it instead of considering that perhaps you are the one who is wrong. Your last comment insinuates that I am dishonest. That's false.
                      $endgroup$
                      – user21820
                      Mar 26 at 9:05











                      1












                      $begingroup$

                      Applying the naive approach of a non-mathematician, to me the expression $z^(5x+1)$ points to a bivariate function,



                      $$f(z,x) = z^(5x+1)$$



                      (because "I see two variables in here"), and with $g(x) = 3$ we have defined



                      $$f(g(x), x) = 3^(5x+1)$$



                      Then



                      $$frac df(g(x),x)dx = frac partial f(g(x),x)partial g(x)cdot frac dg(x)dx + frac partial f(g(x),x)partial xcdot frac dxdx$$



                      $$=frac partial f(g(x),x)partial g(x)cdot 0 + frac partial f(g(x),x)partial xcdot 1 = frac partial f(g(x),x)partial x $$



                      $$=frac partialpartial x left(3^(5x+1)right) $$



                      This appears to be correct, although not useful, since we ended up back in the beginning. Am I doing something wrong here?






                      share|improve this answer









                      $endgroup$












                      • $begingroup$
                        It's unclear whether you know exactly what you're doing. You can't just say "I see two variables in here"! Given any real $x$, the function $f:mathbbR→mathbbR$ defined via $f(y) = x·y$ for $y∈mathbbR$ is a one-input function, not a bivariate function. Yes, the expression "$z^5x+1$" has two variables, so it is meaningful only in a context where both $x,z$ are defined (e.g. $x,z∈mathbbR$ and $z>0$), but that is precisely the true error in the asker's question (see my answer); it did not define $x$, and once you define $x$ you can't reuse it in defining $g$.
                        $endgroup$
                        – user21820
                        Mar 27 at 7:49










                      • $begingroup$
                        Also, it is actually incorrect to write "$frac∂f(g(x),x)∂g(x)$". What you want is $left. frac∂f(t,x)∂t right|_t:=g(x)$. To prove that it is incorrect, consider that if you had wanted $fracd(f(g(x),g(x)))dx$ instead your 'proof' would have included the term "$frac∂f(g(x),g(x))∂g(x)$", which makes the mistake obvious.
                        $endgroup$
                        – user21820
                        Mar 27 at 7:53











                      • $begingroup$
                        @user21820 On your first comment, let $x in mathbbR,; z>0$ and define the bivariate function $f(z,x) = z^(5x+1)$. Further, define $zequiv g(x)$ and also define $g(x) =3$ with $x$ defined as previously. Is there any problem with these definitions? This is not what the OP's student did of course, my answer was a reflection on why $f(z) = z^5x+1$ is not a correct expression, and that the moment you write $z^5x+1$ treating $z$ as a variable, what you can have is a bivariate function since $x$ is already defined (even if implicitly) as a variable.
                        $endgroup$
                        – Alecos Papadopoulos
                        Mar 27 at 9:34










                      • $begingroup$
                        @user21820 Regarding your second comment, I was under the impression that in the left-hand side expression we include a variable of a function only once inside the parenthesis, since the $()$ in $f()$ just lists the variables of the function. When do we want or need to write something like $f(y,y)$, what purpose does it serve?
                        $endgroup$
                        – Alecos Papadopoulos
                        Mar 27 at 9:49










                      • $begingroup$
                        You didn't understand my first comment; please read the second sentence again, which is a counter-example to your "I see two variables" thinking. You also don't seem to understand rigorous notation in your last comment. You say "the () in f() just lists the variables of the function", but you didn't even do that; you wrote "$f(g(x),x)$"! Furthermore, you suggest (falsely) that there is no purpose in writing something like "$f(y,y)$". No, it is not only legitimate but also very useful to be able to evaluate a two-input function along its 'diagonal'.
                        $endgroup$
                        – user21820
                        Mar 27 at 11:25















                      1












                      $begingroup$

                      Applying the naive approach of a non-mathematician, to me the expression $z^(5x+1)$ points to a bivariate function,



                      $$f(z,x) = z^(5x+1)$$



                      (because "I see two variables in here"), and with $g(x) = 3$ we have defined



                      $$f(g(x), x) = 3^(5x+1)$$



                      Then



                      $$frac df(g(x),x)dx = frac partial f(g(x),x)partial g(x)cdot frac dg(x)dx + frac partial f(g(x),x)partial xcdot frac dxdx$$



                      $$=frac partial f(g(x),x)partial g(x)cdot 0 + frac partial f(g(x),x)partial xcdot 1 = frac partial f(g(x),x)partial x $$



                      $$=frac partialpartial x left(3^(5x+1)right) $$



                      This appears to be correct, although not useful, since we ended up back in the beginning. Am I doing something wrong here?






                      share|improve this answer









                      $endgroup$












                      • $begingroup$
                        It's unclear whether you know exactly what you're doing. You can't just say "I see two variables in here"! Given any real $x$, the function $f:mathbbR→mathbbR$ defined via $f(y) = x·y$ for $y∈mathbbR$ is a one-input function, not a bivariate function. Yes, the expression "$z^5x+1$" has two variables, so it is meaningful only in a context where both $x,z$ are defined (e.g. $x,z∈mathbbR$ and $z>0$), but that is precisely the true error in the asker's question (see my answer); it did not define $x$, and once you define $x$ you can't reuse it in defining $g$.
                        $endgroup$
                        – user21820
                        Mar 27 at 7:49










                      • $begingroup$
                        Also, it is actually incorrect to write "$frac∂f(g(x),x)∂g(x)$". What you want is $left. frac∂f(t,x)∂t right|_t:=g(x)$. To prove that it is incorrect, consider that if you had wanted $fracd(f(g(x),g(x)))dx$ instead your 'proof' would have included the term "$frac∂f(g(x),g(x))∂g(x)$", which makes the mistake obvious.
                        $endgroup$
                        – user21820
                        Mar 27 at 7:53











                      • $begingroup$
                        @user21820 On your first comment, let $x in mathbbR,; z>0$ and define the bivariate function $f(z,x) = z^(5x+1)$. Further, define $zequiv g(x)$ and also define $g(x) =3$ with $x$ defined as previously. Is there any problem with these definitions? This is not what the OP's student did of course, my answer was a reflection on why $f(z) = z^5x+1$ is not a correct expression, and that the moment you write $z^5x+1$ treating $z$ as a variable, what you can have is a bivariate function since $x$ is already defined (even if implicitly) as a variable.
                        $endgroup$
                        – Alecos Papadopoulos
                        Mar 27 at 9:34










                      • $begingroup$
                        @user21820 Regarding your second comment, I was under the impression that in the left-hand side expression we include a variable of a function only once inside the parenthesis, since the $()$ in $f()$ just lists the variables of the function. When do we want or need to write something like $f(y,y)$, what purpose does it serve?
                        $endgroup$
                        – Alecos Papadopoulos
                        Mar 27 at 9:49










                      • $begingroup$
                        You didn't understand my first comment; please read the second sentence again, which is a counter-example to your "I see two variables" thinking. You also don't seem to understand rigorous notation in your last comment. You say "the () in f() just lists the variables of the function", but you didn't even do that; you wrote "$f(g(x),x)$"! Furthermore, you suggest (falsely) that there is no purpose in writing something like "$f(y,y)$". No, it is not only legitimate but also very useful to be able to evaluate a two-input function along its 'diagonal'.
                        $endgroup$
                        – user21820
                        Mar 27 at 11:25













                      1












                      1








                      1





                      $begingroup$

                      Applying the naive approach of a non-mathematician, to me the expression $z^(5x+1)$ points to a bivariate function,



                      $$f(z,x) = z^(5x+1)$$



                      (because "I see two variables in here"), and with $g(x) = 3$ we have defined



                      $$f(g(x), x) = 3^(5x+1)$$



                      Then



                      $$frac df(g(x),x)dx = frac partial f(g(x),x)partial g(x)cdot frac dg(x)dx + frac partial f(g(x),x)partial xcdot frac dxdx$$



                      $$=frac partial f(g(x),x)partial g(x)cdot 0 + frac partial f(g(x),x)partial xcdot 1 = frac partial f(g(x),x)partial x $$



                      $$=frac partialpartial x left(3^(5x+1)right) $$



                      This appears to be correct, although not useful, since we ended up back in the beginning. Am I doing something wrong here?






                      share|improve this answer









                      $endgroup$



                      Applying the naive approach of a non-mathematician, to me the expression $z^(5x+1)$ points to a bivariate function,



                      $$f(z,x) = z^(5x+1)$$



                      (because "I see two variables in here"), and with $g(x) = 3$ we have defined



                      $$f(g(x), x) = 3^(5x+1)$$



                      Then



                      $$frac df(g(x),x)dx = frac partial f(g(x),x)partial g(x)cdot frac dg(x)dx + frac partial f(g(x),x)partial xcdot frac dxdx$$



                      $$=frac partial f(g(x),x)partial g(x)cdot 0 + frac partial f(g(x),x)partial xcdot 1 = frac partial f(g(x),x)partial x $$



                      $$=frac partialpartial x left(3^(5x+1)right) $$



                      This appears to be correct, although not useful, since we ended up back in the beginning. Am I doing something wrong here?







                      share|improve this answer












                      share|improve this answer



                      share|improve this answer










                      answered Mar 26 at 22:31









                      Alecos PapadopoulosAlecos Papadopoulos

                      976412




                      976412











                      • $begingroup$
                        It's unclear whether you know exactly what you're doing. You can't just say "I see two variables in here"! Given any real $x$, the function $f:mathbbR→mathbbR$ defined via $f(y) = x·y$ for $y∈mathbbR$ is a one-input function, not a bivariate function. Yes, the expression "$z^5x+1$" has two variables, so it is meaningful only in a context where both $x,z$ are defined (e.g. $x,z∈mathbbR$ and $z>0$), but that is precisely the true error in the asker's question (see my answer); it did not define $x$, and once you define $x$ you can't reuse it in defining $g$.
                        $endgroup$
                        – user21820
                        Mar 27 at 7:49










                      • $begingroup$
                        Also, it is actually incorrect to write "$frac∂f(g(x),x)∂g(x)$". What you want is $left. frac∂f(t,x)∂t right|_t:=g(x)$. To prove that it is incorrect, consider that if you had wanted $fracd(f(g(x),g(x)))dx$ instead your 'proof' would have included the term "$frac∂f(g(x),g(x))∂g(x)$", which makes the mistake obvious.
                        $endgroup$
                        – user21820
                        Mar 27 at 7:53











                      • $begingroup$
                        @user21820 On your first comment, let $x in mathbbR,; z>0$ and define the bivariate function $f(z,x) = z^(5x+1)$. Further, define $zequiv g(x)$ and also define $g(x) =3$ with $x$ defined as previously. Is there any problem with these definitions? This is not what the OP's student did of course, my answer was a reflection on why $f(z) = z^5x+1$ is not a correct expression, and that the moment you write $z^5x+1$ treating $z$ as a variable, what you can have is a bivariate function since $x$ is already defined (even if implicitly) as a variable.
                        $endgroup$
                        – Alecos Papadopoulos
                        Mar 27 at 9:34










                      • $begingroup$
                        @user21820 Regarding your second comment, I was under the impression that in the left-hand side expression we include a variable of a function only once inside the parenthesis, since the $()$ in $f()$ just lists the variables of the function. When do we want or need to write something like $f(y,y)$, what purpose does it serve?
                        $endgroup$
                        – Alecos Papadopoulos
                        Mar 27 at 9:49










                      • $begingroup$
                        You didn't understand my first comment; please read the second sentence again, which is a counter-example to your "I see two variables" thinking. You also don't seem to understand rigorous notation in your last comment. You say "the () in f() just lists the variables of the function", but you didn't even do that; you wrote "$f(g(x),x)$"! Furthermore, you suggest (falsely) that there is no purpose in writing something like "$f(y,y)$". No, it is not only legitimate but also very useful to be able to evaluate a two-input function along its 'diagonal'.
                        $endgroup$
                        – user21820
                        Mar 27 at 11:25
















                      • $begingroup$
                        It's unclear whether you know exactly what you're doing. You can't just say "I see two variables in here"! Given any real $x$, the function $f:mathbbR→mathbbR$ defined via $f(y) = x·y$ for $y∈mathbbR$ is a one-input function, not a bivariate function. Yes, the expression "$z^5x+1$" has two variables, so it is meaningful only in a context where both $x,z$ are defined (e.g. $x,z∈mathbbR$ and $z>0$), but that is precisely the true error in the asker's question (see my answer); it did not define $x$, and once you define $x$ you can't reuse it in defining $g$.
                        $endgroup$
                        – user21820
                        Mar 27 at 7:49










                      • $begingroup$
                        Also, it is actually incorrect to write "$frac∂f(g(x),x)∂g(x)$". What you want is $left. frac∂f(t,x)∂t right|_t:=g(x)$. To prove that it is incorrect, consider that if you had wanted $fracd(f(g(x),g(x)))dx$ instead your 'proof' would have included the term "$frac∂f(g(x),g(x))∂g(x)$", which makes the mistake obvious.
                        $endgroup$
                        – user21820
                        Mar 27 at 7:53











                      • $begingroup$
                        @user21820 On your first comment, let $x in mathbbR,; z>0$ and define the bivariate function $f(z,x) = z^(5x+1)$. Further, define $zequiv g(x)$ and also define $g(x) =3$ with $x$ defined as previously. Is there any problem with these definitions? This is not what the OP's student did of course, my answer was a reflection on why $f(z) = z^5x+1$ is not a correct expression, and that the moment you write $z^5x+1$ treating $z$ as a variable, what you can have is a bivariate function since $x$ is already defined (even if implicitly) as a variable.
                        $endgroup$
                        – Alecos Papadopoulos
                        Mar 27 at 9:34










                      • $begingroup$
                        @user21820 Regarding your second comment, I was under the impression that in the left-hand side expression we include a variable of a function only once inside the parenthesis, since the $()$ in $f()$ just lists the variables of the function. When do we want or need to write something like $f(y,y)$, what purpose does it serve?
                        $endgroup$
                        – Alecos Papadopoulos
                        Mar 27 at 9:49










                      • $begingroup$
                        You didn't understand my first comment; please read the second sentence again, which is a counter-example to your "I see two variables" thinking. You also don't seem to understand rigorous notation in your last comment. You say "the () in f() just lists the variables of the function", but you didn't even do that; you wrote "$f(g(x),x)$"! Furthermore, you suggest (falsely) that there is no purpose in writing something like "$f(y,y)$". No, it is not only legitimate but also very useful to be able to evaluate a two-input function along its 'diagonal'.
                        $endgroup$
                        – user21820
                        Mar 27 at 11:25















                      $begingroup$
                      It's unclear whether you know exactly what you're doing. You can't just say "I see two variables in here"! Given any real $x$, the function $f:mathbbR→mathbbR$ defined via $f(y) = x·y$ for $y∈mathbbR$ is a one-input function, not a bivariate function. Yes, the expression "$z^5x+1$" has two variables, so it is meaningful only in a context where both $x,z$ are defined (e.g. $x,z∈mathbbR$ and $z>0$), but that is precisely the true error in the asker's question (see my answer); it did not define $x$, and once you define $x$ you can't reuse it in defining $g$.
                      $endgroup$
                      – user21820
                      Mar 27 at 7:49




                      $begingroup$
                      It's unclear whether you know exactly what you're doing. You can't just say "I see two variables in here"! Given any real $x$, the function $f:mathbbR→mathbbR$ defined via $f(y) = x·y$ for $y∈mathbbR$ is a one-input function, not a bivariate function. Yes, the expression "$z^5x+1$" has two variables, so it is meaningful only in a context where both $x,z$ are defined (e.g. $x,z∈mathbbR$ and $z>0$), but that is precisely the true error in the asker's question (see my answer); it did not define $x$, and once you define $x$ you can't reuse it in defining $g$.
                      $endgroup$
                      – user21820
                      Mar 27 at 7:49












                      $begingroup$
                      Also, it is actually incorrect to write "$frac∂f(g(x),x)∂g(x)$". What you want is $left. frac∂f(t,x)∂t right|_t:=g(x)$. To prove that it is incorrect, consider that if you had wanted $fracd(f(g(x),g(x)))dx$ instead your 'proof' would have included the term "$frac∂f(g(x),g(x))∂g(x)$", which makes the mistake obvious.
                      $endgroup$
                      – user21820
                      Mar 27 at 7:53





                      $begingroup$
                      Also, it is actually incorrect to write "$frac∂f(g(x),x)∂g(x)$". What you want is $left. frac∂f(t,x)∂t right|_t:=g(x)$. To prove that it is incorrect, consider that if you had wanted $fracd(f(g(x),g(x)))dx$ instead your 'proof' would have included the term "$frac∂f(g(x),g(x))∂g(x)$", which makes the mistake obvious.
                      $endgroup$
                      – user21820
                      Mar 27 at 7:53













                      $begingroup$
                      @user21820 On your first comment, let $x in mathbbR,; z>0$ and define the bivariate function $f(z,x) = z^(5x+1)$. Further, define $zequiv g(x)$ and also define $g(x) =3$ with $x$ defined as previously. Is there any problem with these definitions? This is not what the OP's student did of course, my answer was a reflection on why $f(z) = z^5x+1$ is not a correct expression, and that the moment you write $z^5x+1$ treating $z$ as a variable, what you can have is a bivariate function since $x$ is already defined (even if implicitly) as a variable.
                      $endgroup$
                      – Alecos Papadopoulos
                      Mar 27 at 9:34




                      $begingroup$
                      @user21820 On your first comment, let $x in mathbbR,; z>0$ and define the bivariate function $f(z,x) = z^(5x+1)$. Further, define $zequiv g(x)$ and also define $g(x) =3$ with $x$ defined as previously. Is there any problem with these definitions? This is not what the OP's student did of course, my answer was a reflection on why $f(z) = z^5x+1$ is not a correct expression, and that the moment you write $z^5x+1$ treating $z$ as a variable, what you can have is a bivariate function since $x$ is already defined (even if implicitly) as a variable.
                      $endgroup$
                      – Alecos Papadopoulos
                      Mar 27 at 9:34












                      $begingroup$
                      @user21820 Regarding your second comment, I was under the impression that in the left-hand side expression we include a variable of a function only once inside the parenthesis, since the $()$ in $f()$ just lists the variables of the function. When do we want or need to write something like $f(y,y)$, what purpose does it serve?
                      $endgroup$
                      – Alecos Papadopoulos
                      Mar 27 at 9:49




                      $begingroup$
                      @user21820 Regarding your second comment, I was under the impression that in the left-hand side expression we include a variable of a function only once inside the parenthesis, since the $()$ in $f()$ just lists the variables of the function. When do we want or need to write something like $f(y,y)$, what purpose does it serve?
                      $endgroup$
                      – Alecos Papadopoulos
                      Mar 27 at 9:49












                      $begingroup$
                      You didn't understand my first comment; please read the second sentence again, which is a counter-example to your "I see two variables" thinking. You also don't seem to understand rigorous notation in your last comment. You say "the () in f() just lists the variables of the function", but you didn't even do that; you wrote "$f(g(x),x)$"! Furthermore, you suggest (falsely) that there is no purpose in writing something like "$f(y,y)$". No, it is not only legitimate but also very useful to be able to evaluate a two-input function along its 'diagonal'.
                      $endgroup$
                      – user21820
                      Mar 27 at 11:25




                      $begingroup$
                      You didn't understand my first comment; please read the second sentence again, which is a counter-example to your "I see two variables" thinking. You also don't seem to understand rigorous notation in your last comment. You say "the () in f() just lists the variables of the function", but you didn't even do that; you wrote "$f(g(x),x)$"! Furthermore, you suggest (falsely) that there is no purpose in writing something like "$f(y,y)$". No, it is not only legitimate but also very useful to be able to evaluate a two-input function along its 'diagonal'.
                      $endgroup$
                      – user21820
                      Mar 27 at 11:25

















                      draft saved

                      draft discarded
















































                      Thanks for contributing an answer to Mathematics Educators Stack Exchange!


                      • Please be sure to answer the question. Provide details and share your research!

                      But avoid


                      • Asking for help, clarification, or responding to other answers.

                      • Making statements based on opinion; back them up with references or personal experience.

                      Use MathJax to format equations. MathJax reference.


                      To learn more, see our tips on writing great answers.




                      draft saved


                      draft discarded














                      StackExchange.ready(
                      function ()
                      StackExchange.openid.initPostLogin('.new-post-login', 'https%3a%2f%2fmatheducators.stackexchange.com%2fquestions%2f15366%2fhow-to-explain-whats-wrong-with-this-application-of-the-chain-rule%23new-answer', 'question_page');

                      );

                      Post as a guest















                      Required, but never shown





















































                      Required, but never shown














                      Required, but never shown












                      Required, but never shown







                      Required, but never shown

































                      Required, but never shown














                      Required, but never shown












                      Required, but never shown







                      Required, but never shown







                      Popular posts from this blog

                      Kamusi Yaliyomo Aina za kamusi | Muundo wa kamusi | Faida za kamusi | Dhima ya picha katika kamusi | Marejeo | Tazama pia | Viungo vya nje | UrambazajiKuhusu kamusiGo-SwahiliWiki-KamusiKamusi ya Kiswahili na Kiingerezakuihariri na kuongeza habari

                      Swift 4 - func physicsWorld not invoked on collision? The Next CEO of Stack OverflowHow to call Objective-C code from Swift#ifdef replacement in the Swift language@selector() in Swift?#pragma mark in Swift?Swift for loop: for index, element in array?dispatch_after - GCD in Swift?Swift Beta performance: sorting arraysSplit a String into an array in Swift?The use of Swift 3 @objc inference in Swift 4 mode is deprecated?How to optimize UITableViewCell, because my UITableView lags

                      Access current req object everywhere in Node.js ExpressWhy are global variables considered bad practice? (node.js)Using req & res across functionsHow do I get the path to the current script with Node.js?What is Node.js' Connect, Express and “middleware”?Node.js w/ express error handling in callbackHow to access the GET parameters after “?” in Express?Modify Node.js req object parametersAccess “app” variable inside of ExpressJS/ConnectJS middleware?Node.js Express app - request objectAngular Http Module considered middleware?Session variables in ExpressJSAdd properties to the req object in expressjs with Typescript